krok 2 stomatology€¦ ·  · 2017-10-01krok 2 stomatology (англомовний варiант,...

30
MINISTRY OF PUBLIC HEALTH OF UKRAINE Department of human resources policy, education and science Testing Board Test items for licensing examination Krok 2 STOMATOLOGY Student ID Last name Variant ________________

Upload: phamdan

Post on 25-May-2018

319 views

Category:

Documents


2 download

TRANSCRIPT

Page 1: Krok 2 STOMATOLOGY€¦ ·  · 2017-10-01Krok 2 Stomatology (англомовний варiант, iноземнi студенти) 2017 рiк 1 1. A 27-year-old man complains of

MINISTRY OF PUBLIC HEALTH OF UKRAINE

Department of human resources policy, education and science

Testing Board

Test items for licensing examination

Krok 2

STOMATOLOGY

Student ID Last name

Variant ________________

Page 2: Krok 2 STOMATOLOGY€¦ ·  · 2017-10-01Krok 2 Stomatology (англомовний варiант, iноземнi студенти) 2017 рiк 1 1. A 27-year-old man complains of

ББК 54.1я73УДК 61

Authors of items: Aksonova Ye.A., Amosova L.I., Andrianova I.I., Artiukh V.M., Avdieiev O.V.,Babenko L.M., Bas O.A., Bedeniuk O.A., Beliaieva L.H., Bielikov O.B., Bik Ya.H., Bilous I.V.,Bobkova S.A., Bosa L.F., Chernov D.V., Chyhrynets V.N., Chyzhevsky I.V., Denysova O.H.,Derkach L.Z., Dmytrenko R.R., Dontsova D.O., Dubrovina O.V., Dubrovina-Parus T.A.,Duda K.M., Eismund A.P., Fastovets O.O., Flis P.S., Furdychko A.I., Haiduk R.V., Hanchev K.S.,Herasym L.M., Hirchak H.V., Holik V.P., Holovko N.V., Holubieva I.M., Honcharenko V.A.,Honcharuk L.V., Hordiychuk M.O., Horlenko O.V., Hrad A.O., Hrekuliak V.V., Hrynovets V.S.,Hubanova O.I., Hurzhiy O.V., Ivanitska O.V., Ivchenko N.A., Kalinichenko Yu.A., Kaskova L.F.,Katurova H.F., Kharchenko O.I., Kharkov L.V., Kobryn O.P., Kolesnyk V.M., Kolomeichuk B.Ya.,Konovalov M.F., Kosarieva L.I., Koval O.V., Kovalchuk L.P., Kril A.Y., Krupnyk N.M.,Kryzhanivska O.O., Kuchyrka L.I., Kutelmakh O.I., Kyrychenko V.M., Kyryliuk L.M.,Kyryliuk M.I., Larionov I.M., Larionova L.V., Lokes K.P., Lunhu V.I., Lysenko Yu.H.,Lysiuk S.V., Malakhovska A.O., Marchenko N.V., Mazurina I.O., Mikhalova A.O., Miziuk L.V.,Moiseitseva L.O., Morozova M.M., Muzychina H.A., Nemish T.Yu., Nesyn O.F.,Odzhubeiska O.D., Onyshchenko S.I., Orlovsky V.O., Ostafiychuk M.O., Ozhohan Z.R.,Palis S.Yu., Pantus A.V., Parasochkina V.V., Pasechnyk A.M., Pidlubna L.S., Piuryk V.P.,Posolenyk L.Ya., Prodanchuk A.I., Prots H.B., Raida A.I., Riaboshapko O.A., Riznyk S.S.,Romanenko I.H., Rozumenko O.P., Ruzin H.P., Samsonov O.V., Senchenko O.M.,Shcherbyna I.M., Shmat S.M., Shubladze H.K., Shuvalov S.M., Shuturminsky V.H., Siry O.M.,Stavytsky S.O., Strelchenia T.M., Svirchkov V.N., Sydorchuk S.H., Sydorenko I.V., Sydorova A.I.,Sylenko Yu.I., Tereshyna Z.L., Tiuhashkina Ye.H., Tkachenko P.I., Tril S.I., Tsentilo V.H.,Udod O.O., Valchuk O.H., Yatsenko I.V., Yermakova I.D., Yeroshenko A.V., Yevtushenko L.H.,Yudina O.O., Zavoiko L.M. and Committees of professional expertise.

Item reviewers. Bezvushko E.V., Bulbyk O.I., Chyzhevsky I.V., Dmytriieva A.A.,Fastovets O.O., Gerelyuk V.I., Ilenko N.M., Kaskova L.F., Lungu V.I., Muntian L.M.,Novikov V.M., Ostapko O.I., Smagliuk L.V., Solovey S.I., Tril S.I., Tsentylo V.G., Volynets V.M.,Volyak M.N.

The book includes test items for use at licensing integrated examination “Krok 2. Stomatology” andfurther use in teaching.

The book has been developed for students of stomatological faculties and academic staff of highermedical educational establishments.

Approved by Ministry of Public Health of Ukraine as examination and teachingpublication based on expert conclusions (Orders of MPH of Ukraine of14.08.1998 №251, of 27.12.1999 №303, of 16.10.2002 №374, of 29.05.2003 №233).

© Copyright Testing Board.

General InstructionEvery one of these numbered questions or unfinished statements in

this chapter corresponds to answers or statements endings. Choose theanswer (finished statements) that fits best and fill in the circle with thecorresponding Latin letter on the answer sheet.

Page 3: Krok 2 STOMATOLOGY€¦ ·  · 2017-10-01Krok 2 Stomatology (англомовний варiант, iноземнi студенти) 2017 рiк 1 1. A 27-year-old man complains of

Krok 2 Stomatology (англомовний варiант, iноземнi студенти) 2017 рiк 1

1. A 27-year-old man complains of teethmobility in his upper and lower jaws.Objectively: dentition is intact. Centralocclusion is determined. What examinati-on methods should be applied?

A. Analysis of diagnostic models of the jawsB. GnathodynamometryC. MasticatiographyD. X-rayE. Electromyography

2. A 47-year-old man complains of mobilityof the artificial crown on the 36 tooth, whi-ch was made 2 years ago. Objectively: the36 tooth is covered with full metal swagedcrown. Crown decementation and dentindemineralization are observed. What is thecause of such complication?

A. The crown is loose at the tooth cervixB. The crown edge is embedded into thegingival pocketC. There are interdental contactsD. The crown contacts with antagonisticteethE. Useful life of the crown is exceeded

3. A 48-year-old man complains of itchi-ng gums. Objectively: the gums are firm;cold water causes short-time pain in theteeth. X-ray imaging shows osteosclerosisof the alveolar septum: close-meshed bonestructure, the alveolar septum height andintactness of the cortical plates is retained.What is the most likely diagnosis?

A. Initial periodontosisB. Atrophic gingivitisC. Initial periodontitisD. Periodontitis, stage IE. Periodontosis, stage I

4. A 60-year-old man complains of stabbi-ng pain near the root of the tongue on theright, which develops during eating, especi-ally sour food. Objectively: there is a swelli-ng in the right submandibular area. Onpalpation the submandibular gland is denseand enlarged. Excretory duct orifice of theright submandibular gland is dilated andproduces mucopurulent secretion. What isthe most likely diagnosis?

A. Sialolithiasis of the submandibular glandB. Calculous sialadenitis of the sublingualglandC. Acute suppurative lymphadenitisD. Acute sialodochitisE. Adenophlegmon of the right submandi-bular area

5. A 19-year-old woman, an actress, complai-ns of discoloration of her left maxillarycentral incisor. One year ago the pulp ofthis tooth was removed and the tooth was

filled. Gradually the tooth assumed grayishcolor. Objectively the 11 is filled, discolored,stable, painless on percussion. Deep occlusi-on is observed. What part of the clinicalpresentation contraindicates installation ofan all-porcelain crown?

A. Deep occlusionB. Front teeth defects that cannot becorrected with fillingsC. Enamel hypoplasia with tooth deformati-on and discolorationD. Tooth discolorationE. Devitalized teeth defects that cannot becorrected with dental inlays

6. A 18-year-old woman needs a denture.Objectively: the 21 tooth is dark gray incolor, devitalized; orthognathic occlusion isobserved. The tooth is to be covered withplastic crown. What plastic should be usedto make the crown?

A. Sinma-MB. PhtoraxC. Protacryl-MD. BacrylE. Etacryl

7. A 58-year-old patient has made an appoi-ntment to make a denture. Objectively: the22 tooth is firm and intact. The alveolar crestis atrophied; the palate is flat. Removabledenture is to be made. What approachregarding the 22 tooth should be chosen bya dentist?

A. To make a telescopic crownB. To retain the toothC. Removal of tooth pulpD. Tooth extractionE. To make a stump crown

8. A patient needs his 26 tooth extracted.After application of tuberal anaesthesiahe developed general fatigue, nausea, and,later, severe itching and skin rashes. Whatcomplication occurred in the patient?

A. UrticariaB. Anaphylactic shockC. CollapseD. Quincke’s edemaE. Unconsciousness

9. For a 24-year-old woman a Richmondcrown is being made to restore the crownof the central maxillar incisor. The cap iscompleted. What is the next step?

Page 4: Krok 2 STOMATOLOGY€¦ ·  · 2017-10-01Krok 2 Stomatology (англомовний варiант, iноземнi студенти) 2017 рiк 1 1. A 27-year-old man complains of

Krok 2 Stomatology (англомовний варiант, iноземнi студенти) 2017 рiк 2

A. To fit the cap on the tooth stump andplace the post in the root canalB. To solder the post with the capC. To fit the cap and the post to the toothrootD. To make the combination dental crownE. To fixate the tooth with cement

10. A 25-year-old patient complains ofprofuse gingival hemorrhages, pain in theoral cavity, weakness, fatigue, fever upto 38oC. These presentations appeared aweek ago. Objectively: the patient is pale,adynamic. Examination of the oral mucosareveals multiple hemorrhages, friable gums,necrotic areas on the tops of gingival papi-llae, as well as enlarged, soft, painless lymphnodes. The oral mucosal lesion can be asymptom of the following disease:

A. Acute leukemiaB. Chronic leukemiaC. Vincent’s stomatitisD. Hypovitaminosis CE. Intoxication with heavy metal salts

11. A 67-year-old patient complains ofrecurrent erosion on the red border ofthe lower lip. Objectively: the erosion isoval in shape, 0,8х1,3 sm in size, coveredin thin scabs that reveal glossy surfacewith punctate bleeding, when removed.There are atrophic areas of the red borderdetected. Infiltration elements are absent.The submandibular lymph nodes are notenlarged. What is the provisional diagnosis?

A. Manganotti’s abrasive precancerous chei-litisB. Leukoplakia, erosive ulcerative formC. KeratoacanthosisD. Bowen’s diseaseE. Cheilitis glandularis

12. A 53-year-old patient complains of painand clicking in the left temporomandibularjoint. Objectively: the face is symmetrical,palpation of the lateral pterygoid muscles ispainful on the left side. Mouth opening isreduced. Tomography shows the bone outli-ne of joint surfaces to be smooth. Which di-sease of those listed below corresponds withthis clinical presentation?

A. Temporomandibular joint disfunctionB. Rheumatic arthritisC. Deforming arthrosisD. Acute posttraumatic arthritisE. Joint ankylosis

13. A 22-year-old patient complains of a pai-nful swelling in the right parotid gland. Aweek earlier the patient received a cheekabrasion that healed under the purulentcrust. Over the past two days the patienthad been observing progressing pain and

fever up to 38, 6oC. Objectively: there is asoft tissue edema in the right parotid regi-on, the skin is slightly strained, without di-scoloration. There is a dense painful infi-ltration 2,5x3,5 cm large, the skin over itexhibits limited mobility. The mouth can befully opened, the mucous membrane aroundthe orifice of the salivary duct is unchanged,saliva is transparent. What is the most likelydiagnosis?

A. Acute lymphadenitisB. Exacerbation of chronic parotitisC. Abscess of the parotid-masseteric regionD. Acute non-epidemic parotitisE. Epidemic parotitis

14. A 33-year-old woman has been admi-tted to the dentofacial department withcomplaints of pain and edema in the rightsubmandibular region, body temperaturerise up to 39, 5oC. Objectively: the patienthas asymmetric face because of soft tissueedema of the right submandibular region,palpation reveals a dense infiltration, theskin over it is hyperemic, cannot make afold. The 46 tooth has a deep carious cavity.What is the most likely diagnosis?

A. Submandibular phlegmon on the rightB. Acute submandibular sialadenitisC. Acute suppurative periostitis of themandibleD. Acute suppurative submandibularlymphadenitisE. Acute right-sided osteomyelitis of themandible

15. Having recovered from acute respiratorydisease, a patient has made an appointmentwith a dentist. The patient complains ofpain in the gums, foul breath, inability totake food, general weakness. Objectively:the gums are hyperemic, swollen, coveredwith necrotic dirty-gray coating; the gumsbleed when the coating is removed. Microbi-ological study of tissues revealed a greatnumber of cocci, bacilli, fusobacteria, andspirochaete. Specify the drug for etiotropictreatment:

A. MetronidazoleB. GalascorbinumC. Potassium permanganateD. TripsinE. Carotolinum (Betacarotene)

16. A 35-year-old patient complains of itch,burning and edema of lips. These presentati-ons occured a week ago. Objectively: there isreddening of the red border and skin, especi-ally in the area of the mouth corners, thereare also vesicles, scabs, small cracks againstthe background of erythematous affectionof the red border. What is the most likelydiagnosis?

Page 5: Krok 2 STOMATOLOGY€¦ ·  · 2017-10-01Krok 2 Stomatology (англомовний варiант, iноземнi студенти) 2017 рiк 1 1. A 27-year-old man complains of

Krok 2 Stomatology (англомовний варiант, iноземнi студенти) 2017 рiк 3

A. Acute eczematous cheilitisB. Multiform exudative erythemaC. Acute herpetic cheilitisD. Allergic contact cheilitisE. Exudative form of exfoliative cheilitis

17. A 47-year-old patient presents withrounded bone protrusions 0,7-0,8 cm in si-ze on the inner surface of the edentulousmandible in the premolar area. The denturefor this patient should have:

A. Elastic linerB. Kemeny claspsC. Metal baseD. Orifices for the exostosesE. Dentogingival clasps

18. A 78-year-old patient is completelyedentulous. He has been wearing denturesfor 19 years. The patient complains of poorfixation of the upper denture. Objectively:the lower third of face is shortened, thealveolar processes of both jaws are markedlyatrophied, the palate is flat. Mucousmembrane in the denture-supporting areais atrophied. How often should the denturesbe remodelled or restored?

A. Every 3-4 yearsB. Every 6 monthsC. Once a yearD. Every 7 yearsE. Every 10-12 years

19. A 12-year-old boy complains of pai-nful and bleeding gums on his upper jaw.Objectively the gingival margin in the areaof the 13, 12, 11, 21, 22, 23 teeth is swollen,hyperemic, deformed due to overgrowths.Gingival papilla cover the crowns by 1/3 oftheir height, bleed on touch. Upper frontteeth are overcrowded. X-ray shows nopathological changes of the periodontium.What drugs should be administered for topi-cal treatment in the first place?

A. Nonsteroidal antiinflammatory drugsB. SclerosantsC. Steroidal antiinflammatory drugsD. Keratoplastic agentsE. Cytostatic agents

20. A girl is 8 years old. She complainsof impaired mastication. Objectively: onexamination of the oral cavity the cuttingedges of her lower incisors touch the palati-ne mucosa in the frontal area; the upperfrontal teeth overlap with the lower onesby full height of their crowns. On the lowerjaw the occlusal curve of the front teeth ismarkedly concave. Make the provisional di-agnosis:

A. Deep overbiteB. Open biteC. False prognathismD. True prognathismE. Cross bite

21. A 43-year-old woman complains ofmobility and displacement of her upperfront teeth. Objectively: dental formula is

17 16 15 14 13 12 11 21 22 23 24 25 26 2747 46 45 44 43 42 41 31 32 33 34 35 36 37 .

Teeth 12 11 21 22 are slanted towardsthe vestibular side, diastema and tremataare observed, I-II degree teeth mobility isdetected. Select the orthodontic appliancefor correction of teeth misalignment as apart of complex treatment of periodontal di-sease:

A. Palatal plate with vestibular archB. Bynin applianceC. Schwartz applianceD. Katz crownE. Palatal plate with inclined plane

22. A 44-year-old patient consulted a dentalsurgeon about constant acute pain in theupper jaw region on the left that aggravatesduring teeth joining. The pain appeared 3days ago. Objectively: the face is symmetric,mouth opening is not limited. The crownof the 26 tooth is half-decayed. Probingof the carious cavity is painless. Percussi-on of the 26 tooth provokes acute pain.Mucous membrane of the alveolar processis edematic, hyperemic at the level of the 26tooth. The 26 tooth had been treated before.What is your provisional diagnosis?

A. Exacerbation of chronic periodontitis ofthe 26 toothB. Acute suppurative periodontitis of the 26toothC. Acute pulpitis of the 26 toothD. Acute suppurative periostitis of the leftupper jaw extending from the 26 toothE. Periodontitis of the 26, 27, and 28 teeth

23. When a prosthodontist was preparingthe patient’s tooth, the patient had epilepticseizure that was subsequently terminated.What mistake had been made by the doctor?

A. No inquire into the patient anamnesisB. No inquire into the antecedent anamnesisC. No anaesthesiaD. Crude preparationE. Did not decline the appointment

24. Parents of a 3-year-old child report thatthe child suffers from constant pain in theupper front teeth. Objectively: the coronalpart of the 61 tooth is gray and decayed.Probing of the root canal orifice is painfuland accompanied by bleeding. The toothpercussion provokes acute pain. Mucosa is

Page 6: Krok 2 STOMATOLOGY€¦ ·  · 2017-10-01Krok 2 Stomatology (англомовний варiант, iноземнi студенти) 2017 рiк 1 1. A 27-year-old man complains of

Krok 2 Stomatology (англомовний варiант, iноземнi студенти) 2017 рiк 4

hyperemic, edematic and painful. Palpationin the region of the 61 and 62 teeth reveals afistula. What is your provisional diagnosis?

A. Exacerbation of chronic periodontitisB. Acute suppurative periodontitisC. Acute diffuse pulpitisD. Chronic granulating periodontitisE. Exacerbation of chronic pulpitis

25. A 30-year-old patient complains ofa toothache caused by hot and cold sti-muli. The pain irradiates to the ear andtemple. Previously this tooth presented withspontaneous nocturnal toothache. Objecti-vely: on the occlusal surface of the 37 tooththere is a deep carious cavity communicatingat one point with the tooth cavity. Probingat the communication point, as well as coldstimulus, causes acute pain. The pain persi-sts for a long time. Electric pulp test resultis 5 microamperes. What is the most likelydiagnosis?

A. Exacerbation of chronic pulpitisB. Acute diffuse pulpitisC. Exacerbation of chronic periodontitisD. Chronic concrementous pulpitisE. Acute suppurative pulpitis

26. A 52-year-old woman complains of peri-odical appearance of a gingival fistula in thearea of the 15 tooth. The tooth had beentreated 1,5 years ago for caries. Objecti-vely: the 15 tooth is filled. In the root apexprojection there is a fistula; purulent exudatedischarges on pressure. Tooth percussion ispainless. On X-ray: the root canal is not fi-lled, there is a destruction focus with blurredmargins near the root. Make the diagnosis:

A. Chronic granulating periodontitisB. Exacerbation of chronic granulatingperiodontitisC. Chronic fibrous periodontitisD. Periapical cystE. Chronic granulomatous periodontitis

27. A man was diagnosed with hard palateabscess. What approach should be chosenfor abscess dissection?

A. Triangular dissection of the hard palateareaB. Linear dissection parallel to the hardpalate rapheC. Linear dissection perpendicular to thehard palate rapheD. Pus aspiration with a syringeE. Abscess puncture

28. A 32-year-old patient presents withbody temperature of 38, 9oC, general fati-gue, impaired speech, inability to eat. Thiscondition has been recurring for the last 4years in autumn and spring. There are vesi-

cles and erosions covered in grayish fibri-nous coating on the hyperemic and swollenlabial and buccal mucosa. Nikolsky’s sign isnegative. What is the most likely diagnosis?

A. Erythema multiformeB. Pemphigus vulgarisC. Acute herpetic stomatitisD. Nonacantholytic pemphigusE. Dermatitis herpetiformis (Duhring’sdisease)

29. A 56-year-old man complains ofenlarged lower lip, pain induced by hot,sour, salty, and bitter foods, and lips glui-ng together in the morning. The lower liphas been gradually enlarging and developi-ng lumps his whole life. On examination:the lower lip is enlarged. The middle thi-rd of the Klein’s zone presents with severalsmall red dots with openings that dischargeclear drops. The red border is dry and peeli-ng, there are fissures and erosions. What isthe most likely diagnosis?

A. Cheilitis glandularisB. Cheilitis exfoliativaC. Cheilitis actinicaD. Allergic contact cheilitisE. Atopic cheilitis

30. A 57-year-old retired man complainsof attacks of burning pain and rashes onthe skin of his face and oral mucosa onthe right. Anamnesis: a course of radiati-on therapy for treatment of the gastric di-sease, past case of chickenpox. Objecti-vely: along the third branch of the trigemi-nal nerve the skin of the face presents wi-th isolated erosions covered in fibrinouscoating. There are multiple vesicles on thehyperemic and swollen oral mucosa. Right-sided lymphadenitis is observed. What di-agnosis is the most likely?

A. Herpes zosterB. NeuralgiaC. MurrainD. Acute recurrent herpesE. Neuritis

31. A 42-year-old woman complains of acutepain in her lower jaw, teeth mobility, highfever. The condition persists for 2 days. Onclinical examination a doctor diagnosed herwith acute mandibular osteomyelitis. Whattactics regarding the mobile teeth should thedoctor choose?

Page 7: Krok 2 STOMATOLOGY€¦ ·  · 2017-10-01Krok 2 Stomatology (англомовний варiант, iноземнi студенти) 2017 рiк 1 1. A 27-year-old man complains of

Krok 2 Stomatology (англомовний варiант, iноземнi студенти) 2017 рiк 5

A. Extraction of the causative tooth onlyB. Extraction of all mobile teethC. Conservative treatment of the causativetoothD. Conservative treatment of all mobile teethE. Extraction of the causative tooth,trepanation and treatment of all mobileteeth

32. A 7-year-old boy complains of increasedbody temperature, up to 38oC, headache,sore throat. Objectively: there are erosionson the slightly hyperemic mucosa of thesoft palate, anterior palatal bars, and tonsils.The submandibular lymph nodes are slightlyenlarged, painless. Name the causative agentof this disease:

A. Coxsackie virusB. Herpes simplex virusC. Epstein-Barr virusD. Klebs-Loeffler bacillusE. Bordet-Gengou bacillus

33. A 6-year-old boy complains of a cavi-ty in the prevoiusly treated tooth. Objecti-vely: in the 85 tooth there is a carious cavitywithin mantle dentin; the dentin is denseand pigmented; probing of the cavilty floorand walls, thermal stimuli, and percussi-on are painless. Sensitivity of the dentin-enamel junction is observed during the cavi-ty preparation. Make the diagnosis:

A. Chronic median cariesB. Chronic deep cariesC. Acute median cariesD. Acute deep cariesE. Chronic superficial caries

34. A patient consulted a dentist about acosmetic defect in the cervical region ofthe upper and lower canines. Various sti-muli cause no pain. Objectively: there areV-shaped defects on the vestibular surfacein the cervical area of the upper and lowercanines. Their surface is smooth, glossy,hard. There is no reaction to probing andcold stimuli. What treatment should beadministered?

A. Filling of the defectsB. Metal crownsC. Applications with 10% solution of calciumgluconateD. Application of fluorine lacquerE. Medical intervention is unnecessary

35. A boy is 10 years old. He complainsof sloped chin and impaired mastication.Anamnesis states formula feeding. Objecti-vely: corellation of the 6th teeth is of theAngle’s II class. Sagittal fissure is 7 mm.Eschler-Bittner test is positive. What is themost likely diagnosis?

A. Prognathism, distal mandibular di-splacementB. Prognathism, maxillary macrognathiaC. Retrusion of the lower jaw frontal areaD. Progenia, mandibular macrognathiaE. Protrusion of the upper jaw frontal area

36. A 10-month-old child is fussy, refusesto eat. Disease onset was 2 days ago. Thechild is been treated by a pediatrician forpneumonia, receives antibiotics and sulfani-lamides. Objectively: the oral mucosa ishyperemic, swollen; there is whitish coati-ng on the mucosa of the cheeks, lips, softand hard palate; coating removal can causeerosions. Submandibular lymph nodes areenlarged. What is the most likely diagnosis?

A. Acute candidal stomatitisB. Acute herpetic stomatitisC. Geographic tongueD. Allergic contact stomatitisE. Chronic candidal stomatitis

37. A 20-year-old man complains of sharppain in the mouth, increase of bodytemeperature up to 38, 5oC, headache andaching joints, general weakness. The diseaseonset was 3 days ago due to overexposure tocold. Objectively: the red border is coveredwith hemorrhagic scabs, oral mucosa haslarge erosions and ulcers merging witheach other and covered with grayish-whitecoating against the background of diffusehyperemia. Conjunctivitis is observed. Theskin of the forearms has erythematous spots1,5 cm in diameter, with blisters in theircenter. What is the most likely diagnosis?

A. Stevens-Johnson syndromeB. Erythema multiformeC. Drug-induced stomatitisD. Lyell’s syndromeE. Behcet’s syndrome

38. Parents of a 12-year-old child areconcerned about the child having whitespots on the frontal teeth of the upper jaw;the spots appeared half a year ago. Objecti-vely: there are chalky spots detected in thecervical zone vestibular surfaces of the 13,12, 11, 21, 22, and 23 teeth. The enamel inthose spots is dull; probing revealed it tobe pliant and coarse. The anamnesis statesshort-time pain caused by chemical stimuli.What is the provisional diagnosis?

A. Acute initial cariesB. Chronic initial cariesC. Acute superficial cariesD. Systemic hypoplasia of enamelE. Dental fluorosis

39. A 22-year-old woman complains of paincaused by hot food and bursting sensationin the tooth. Half a year ago she presented

Page 8: Krok 2 STOMATOLOGY€¦ ·  · 2017-10-01Krok 2 Stomatology (англомовний варiант, iноземнi студенти) 2017 рiк 1 1. A 27-year-old man complains of

Krok 2 Stomatology (англомовний варiант, iноземнi студенти) 2017 рiк 6

with brief bouts of pain in the night, whi-ch over time increased in duration. Objecti-vely: there is a large carious cavity in the24 tooth, which opens to the dental cavity,deep probing is painful. Electric pulp testis 80 microamperes. What is the most likelydiagnosis?

A. Chronic gangrenous pulpitisB. Chronic concrementous pulpitisC. Chronic hypertrophic pulpitisD. Acute suppurative pulpitisE. Chronic fibrous pulpitis

40. A 14-year-old girl complains of bleedi-ng gums and foul smell from her mouth.Objectively: gingival mucosa is hyperemic,pastose, hemorrhaging. Schiller-Pisarev testis positive. Papillary marginal alveolar indexis 70%. Fedorov-Volodkina Hygiene Indexequals 3. X-ray of the frontal area of jawsdemonstrates retained cortical plate. Makethe diagnosis:

A. Chronic generalized catarrhal gingivitisB. Chronic generalized periodontitisC. Acute generalized catarrhal gingivitisD. Chronic generalized hypertrophic gingivi-tisE. Exacerbation of chronic generalizedperiodontitis

41. A 40-year-old man had his root canalof the 34 tooth filled due to chronic fi-brous periodontitis. Soon the treated placebecame painful. On X-ray the root canal ofthe 34 tooth is filled to the root apex. Whattactics should the dentist choose to managethe pain?

A. To prescribe physiotherapeutic proceduresB. To rinse with antiseptic mouthwashC. To make insicion along the mucogingivalfoldD. To provide conduction anesthesiaE. To provide infiltration anesthesia

42. A 38-year-old patient with chronicgeneralized periodontitis has been referredfor orthopedic treatment. Objectively:dentitions are without gaps, the 12, 11, 21,and 22 teeth are pulpless and exhibit I grademobility. The other teeth are firm. Whatis the most aesthetic dental splint for theanterior teeth?

A. Mamlok’s splintB. Ring splintC. Soldered combined crownsD. Cap splintE. Mouthguard

43. The 40-year-old woman complains ofinability to properly masticate due to theloss of the following lateral teeth: 18, 16,15, 25, 26, 28, 38, 35, 36, 44-46, and 48.

The rest of her teeth present with the I-IIdegree of mobility. Generalized periodonti-tis is observed. What denture constructionwould be optimal in this case?

A. Removable dental splintB. Clasp-retained (bugel) removable partialdentureC. Removable laminar dentureD. Fixed dental bridgeE. Metal-based denture

44. A 55-year-old patient requires a denture.Objectively: Kennedy’s I class dentitiondefect; the 16, 17, 18, 26, 27, and 28 teethare missing. The patient presents with fi-xed occlusion. The 15 and 25 teeth havelow crowns with poor anatomic contours,intact. Clasp-retained (bugel) removablepartial denture is being made for the pati-ent. What fixation system would be optimalin this case?

A. Telescopic fixationB. AttachmentsC. Roach clasp (clammer)D. Aker-Roach combined clasp (clammer)E. Continuous clasp (clammer)

45. A patient needs a clasp-retained (bugel)removable partial denture. It is plannedto study the jaw model by means of aparallelometer in order to determine therequired depth of the undercuts on theabutment teeth. Specify the length of themeasuring rods used for this purpose:

A. 0,25 0,50 0,75B. 0,15 0,40 0,65C. 0,20 0,45 0,70D. 0,30 0,55 0,80E. 0,35 0,60 0,85

46. In a prostodontic clinic a partial lami-nar denture for the upper jaw is being madefor a 53-year-old patient. Objectively: dentalformula is 14, 13, 12, 11, 21, 22, 23, 24, 27.The teeth are firm, clinical crowns are tallwith pronounced equator. X-ray shows noperiapical changes in the periodontium ofthe abutment teeth. What clasp fixation isoptimal for this patient?

A. PlanarB. SagittalC. DiagonalD. TransversalE. Point

47. A 20-year-old man complains of missi-ng tooth on the upper right jaw, aestheticdefect. Objectively: the 12 tooth is absent,adjacent teeth are intact, firm, with disti-nct anatomical shape and tall crowns. Directocclusion is observed. During the interviewthe patient was found out to have congeni-

Page 9: Krok 2 STOMATOLOGY€¦ ·  · 2017-10-01Krok 2 Stomatology (англомовний варiант, iноземнi студенти) 2017 рiк 1 1. A 27-year-old man complains of

Krok 2 Stomatology (англомовний варiант, iноземнi студенти) 2017 рiк 7

tal heart disease. What denture constructionwould be optimal in this case?

A. Adgesive dental bridgeB. Plastic dental bridgeC. Porcelain-fused-to-metal dental bridgewith 14 and 13 abutment teethD. Plastic-fused-to-metal dental bridgeE. Swaged-soldered dental bridge

48. The 15 tooth must be extracted. Thetooth crown is retained. What instrumentshould be used in this case?

A. Forceps with S-shaped handlesB. Straight forcepsC. Bayonet forcepsD. Left-sided forceps with S-shaped handlesE. Right-sided forceps with S-shaped handles

49. A 7-year-old child complains of pain andswelling in the left submandibular region.The swelling in this region developed 2 daysago. Objectively: the child is in a satisfactorycondition, body temperature is of 37, 3oC.Face is asymmetrical due to the soft tissueswelling in the left submandibular region.Palpation reveals a round formation 2x2 cmin size. The formation is mobile, painful,unattached to the skin. The 74 tooth is di-scolored, percussion is painful. What is theprovisional diagnosis?

A. Acute serous odontogenic lymphadenitisof the left submandibular regionB. Acute serous nonodontogeniclymphadenitis of the left submandibularregionC. Acute suppurative odontogeniclymphadenitis of the left submandibularregionD. Phlegmonous adenitis of the rightsubmandibular regionE. Lateral cervical cyst

50. A 32-year-old woman complains oftumor-like growth in the mucosa of her leftcheek. Locally: buccal mucosa is of normalcolor. In the distal area there is a roundedelongated growth, soft and elastic, attachedto a pedicle sized 0,5х1,5 cm. Make theprovisional diagnosis:

A. PapillomaB. LipomaC. HemangiomaD. Pleomorphic adenomaE. Fibroma

51. A 52-year-old patient complains of painand swelling in the right parotid region.These manifestations have been presentfor about 2 years. Over the last month theswelling has enlarged, pain has intensifi-ed. Objectively: the face is asymmetric dueto the dense infiltrate in the right parotid

region. The poorly circumscribed, painfulformation infiltrates the surrounding tissues.At the right side of neck in front and behi-nd the sternocleidomastoid muscle there areenlarged, dense, mobile lymph nodes. Theright naso-buccal groove is flattened, thecorner of the mouth is downturned. Themouth opens freely. The are pronouncedsymptoms of the right facial nerve paresis.What disease can be suspected?

A. Adenocarcinoma of the parotid salivaryglandB. Chronic parotitisC. Actinomycosis of the parotid-massetericregionD. Chronic lymphadenitisE. Pleomorphic adenoma of the parotidgland

52. A 57-year-old woman came to a denti-st for extraction of the 34 tooth due toexacerbation of chronic periodontitis. Whatinstrument would be optimal for toothextraction in the given case?

A. Beak-shaped non-crushing forcepsB. Beak-shaped crushing forcepsC. Beak-shaped curved forcepsD. Straight elevatorE. Curved elevators

53. A 35-year-old patient has been di-agnosed with chronic median caries of the36 tooth. There is a Black’s class II cavityaffecting masticatory surface. What materialshould be chosen for the tooth filling?

A. Light-cure microhybrid compositeB. Glass ionomer cementC. Silicophosphate cementD. Light-cure fluid compositeE. Light-cure microfilled composite

54. A cast clasp-retained (bugel) removablepartial denture is being made for a 58-year-old patient. Impressions are made, centricjaw relation is determined, plaster casts areobtained. What is the next stage?

A. Examination of the working model with aparallelometerB. Transfer of denture frame pattern to theworking modelC. Wax modelling of the denture frameD. Duplication of the working modelE. Marking the border seal

55. A 13-year-old boy complains of generalweakness, high body temperature up to39oC, lack of appetite, constant pain in thebody of the lower jaw. Objectively: observedis significant asymmetry of the face causedby soft tissues swelling in the left buccaland submandibular areas. Mouth openingis reduced. Intraoral examination revealed

Page 10: Krok 2 STOMATOLOGY€¦ ·  · 2017-10-01Krok 2 Stomatology (англомовний варiант, iноземнi студенти) 2017 рiк 1 1. A 27-year-old man complains of

Krok 2 Stomatology (англомовний варiант, iноземнi студенти) 2017 рiк 8

the following: the 34, 35, 36, and 37 teethare mobile; teeth percussion is painful.The crown of the 36 tooth is completelydestroyed. The mucosa of those teeth ishyperemic and painful when palpated. Muff-like enlargement of the lower jaw alveolarprocess is detected. What is the most likelydiagnosis?

A. Acute mandibular odontogenicosteomyelitisB. Acute mandibular hematogenousosteomyelitisC. Acute mandibular odontogenic suppurati-ve periostitisD. Ewing’s sarcomaE. Abscess of the right submandibular area

56. What denture constructions should bechosen in the cases of multiple adentia duri-ng the initial period of occlusion change?

A. Removable partial dentureB. Dental bridgeC. Clasp-retained (bugel) removable partialdentureD. No denture is necessaryE. Removable complete denture

57. On objective examination a 59-year-oldman with the edentulous mandible presentswith bone protrusions and mobile areas ofthe alveolar crest. To ensure proper fixationof the denture and even load distributionthe following functional impression shouldbe made:

A. DifferentiatedB. Complete anatomicalC. CompressionD. DecompressionE. Combined

58. A 49-year-old woman complains ofcosmetic defect of the 11, 21, and 22 teeth,which developed over a year ago. Objecti-vely: on the vestibular surface at the equatorof the 11, 21, and 22 teeth there are shallowcup-shaped enamel defects that are dense onprobing. Cold water induces no pain. Makethe provisional diagnosis:

A. Enamel erosionB. Cuneiform defectC. Superficial cariesD. HypoplasiaE. Fluorosis

59. A 47-year-old patient complains of aburning sensation and pain in the mouth.Objectively: on the mucous membrane ofcheeks along the line of teeth contact and inthe corners of the mouth there are multi-ple polygonal bright red erosions 1,0-1,5cm in diameter located on the hyperkerati-nized plaque and opaque whitish mucosa.

Cytological analysis revealed keratinizingepithelial cells. What is the most likely di-agnosis?

A. Leukoplakia, erosive formB. Lichen ruber planus, erosive formC. Erythema multiformeD. Secondary syphilisE. Lupus erythematosus, erosive form

60. A 28-year-old man complains of pain inthe infraorbital and parotid region on theleft. On examination: hemorrhage occurs inthe lower eyelid and conjunctiva of the lefteye, there are signs of crepitation and stepdeformity of the eyesocket lower edge. Themouth opens by 1 cm. Make the diagnosis:

A. Zygomatic bone fractureB. Malar arch fractureC. Left articular process fractureD. Traumatic arthritis of the temporomandi-bular jointE. Hematoma of the infraorbital region

61. A child is 8 years old. There are complai-nts of congested upper incisors. Objectively:the first molars closure is of Angle’s I class,frontal overbite is orthognathic. The 12 and22 teeth erupt palatinally with space defici-ency of 2/3 of the tooth crown. The 11 and 21teeth are 10 mm each in cross-section. Thechild has inherited father’s facial type withprognathism and macrodontia of the centralincisors. Choose the preventive treatment,considering this hereditary pathology:

A. Hotz serial extraction to reduce the dentalarchB. Jaw expansion to provide the space for the12 and 21 teethC. Massage of the 12 and 21 teeth area tostimulate their eruptionD. Extraction of the 12 and 21 teeth to reducethe dental archE. Filing down of the 11 and 21 approximalsurfaces to provide the space for the 12 and22 teeth

62. A 65-year-old woman complains of aneoplasm in the area of the nasolabial foldon the left, which appeared one month ago.Objectively: there is a gray neoplasm onthe skin of the nasolabial fold on the left,markedly keratotic, 3,0х0,5х0,3 cm in si-ze. Neoplastic base is painless, dense, andelastic. What is the most likely pathologythat results in such clinical presentation?

A. Cutaneous horn of the left nasolabial foldB. Common wart of the left nasolabial foldC. Senile keratosis of the left nasolabial foldD. Keratoacanthoma of the left nasolabialfoldE. Lupus

Page 11: Krok 2 STOMATOLOGY€¦ ·  · 2017-10-01Krok 2 Stomatology (англомовний варiант, iноземнi студенти) 2017 рiк 1 1. A 27-year-old man complains of

Krok 2 Stomatology (англомовний варiант, iноземнi студенти) 2017 рiк 9

63. A 55-year-old patient consulted a denti-st about a rounded tumor-like formationabout 1 cm in diameter located within thered border of his lower lip. Objectively: thetumor-like formation protrudes about 5 mmabove the red border, is dense and grayish-red. The surface of the formation is coveredwith thin scales that can hardly be removed.What is the most likely diagnosis?

A. Verrucous precancer of the red borderB. Manganotti’s abrasive precancerous chei-litisC. Precancerous limited hyperkeratosis ofthe red borderD. Bowen’s diseaseE. Erythroplasia of Queyrat

64. A 38-year-old woman complains ofburning pain in her lips and angles of hermouth, their dryness. Anamnesis statesthat she has been suffering from diabetesmellitus for the last 8 years. Objecti-vely: the red border is dry, congestivelyhyperemic, covered in scales of variable si-ze. In the angles of the mouth there arefissures covered in white coating, the skinis macerated. What ointment should beprescribed for topical treatment in the gi-ven case?

A. ClotrimazolB. InterferonC. PrednisoloneD. LanolinE. Erythromycin

65. A 35-year-old woman complains of lipsenlargement. The first incident occurred oneyear ago, when she developed lip edema thatabated quickly, but the lips remained slightlyenlarged. Three days ago after overexposureto cold her lips enlarged again. Objectively:ptosis, upper and lower lips are markedlyenlarged, more on the left, soft, elastic, andpainless on palpation; no impressions on thelip surface are left after pressing it with afinger. The tongue is swollen, with tuberoussurface and folds on its back. What is themost likely diagnosis?

A. Melkersson-Rosenthal syndromeB. Miescher granulomatous cheilitisC. Quincke’s edemaD. Achard’s syndromeE. Meige’s trophedema

66. A 5-year-old child has bad habit of sucki-ng on his tongue. At the front area there isa small vertical fissure up to 2 mm in size.Neutral closure is observed in the lateralareas of the jaws. The child is diagnosed wi-th open traumatic bite of the I degree. Avestibulo-buccal shield was prescribed fortreatment. What is the function of the appli-ance in the given case?

A. Treatment and preventionB. PreventionC. RetentionD. PassiveE. Treatment

67. A 40-year-old man, a chemical industryworker, notes the development of sour tasteof the mouth, pain response to thermal andchemical stimuli. On examination: on thevestibular surface and cutting edge of thefront teeth there are enamel defects wi-th uneven scalloped margins, chalk-like incolor. Make the diagnosis:

A. Acidic necrosis of enamelB. Superficial cariesC. Enamel hypoplasia (erosive form)D. Fluorosis (erosive form)E. Median caries

68. A patient with post-resection upper jawdefect that invades the nasal cavity has cometo a prosthodontic clinic. What denture isrecommended in the given case?

A. Replacement denture with obturatingelementB. Floating obturatorC. MouthguardD. Forming dentureE. Replacement denture

69. An injured patient complains of reducedopening of the mouth, nose bleeding, skinnumbness in the infraorbital and lowereyelid region. Objectively: there is facedeformation due to the depression of softtissues in the left cheekbone region, stepdeformity in the middle part of the inferi-or margin of the left orbit and in the areaof the zygomatic alveolar crest. What is themost likely diagnosis?

A. Zygomatic bone fracture with di-splacement of the bone fragmentsB. Fracture of the right zygomatic bonewithout displacement of the bone fragmentsC. Le Fort I fracture of maxillaD. Le Fort II fracture of maxillaE. Fracture of the malar arch

70. A 30-year-old woman came to a dentistwith complaints of a slightly bleeding ”sore”on her lower lip, which is located at its medi-an and bisects the lip into two even parts. Onpalpation the lip is swollen and slightly pai-nful. What diagnosis corresponds with thegiven clinical presentation?

Page 12: Krok 2 STOMATOLOGY€¦ ·  · 2017-10-01Krok 2 Stomatology (англомовний варiант, iноземнi студенти) 2017 рiк 1 1. A 27-year-old man complains of

Krok 2 Stomatology (англомовний варiант, iноземнi студенти) 2017 рiк 10

A. Chronic labial fissureB. Tappeiner’s leukoplakiaC. Erosive-ulcerative leukoplakiaD. Lichen ruber planus, erosive-ulcerativeformE. Meteorological cheilitis

71. A 30-year-old patient needs to have his26 tooth extracted because of exacerbati-on of chronic periodontitis. Objectively: thecrown of the 26 tooth is decayed by 1/3.What forceps can be used for this toothextraction?

A. S-shaped forceps with a projecting tip onthe left beakB. S-shaped forceps with a projecting tip onthe right beakC. Straight forcepsD. Straight elevatorE. S-shaped forceps without projecting tips

72. A 62-year-old patient came to a dentalclinic with complaints of facial swelling, painin the lower left jaw, and numb lower lip. Onclinical examination he was diagnosed withfracture of the body of mandible on the left,edentulous jaws, microstomia. Choose theoptimal construction:

A. Limberg’s dental splintB. Weber’s dental splintC. Guning-Port’s dental splintD. Elbrecht’s dental splintE. Vankevych dental splint

73. Prior to dental treatment a 13-year-oldpatient had been administered anaesthesia.The patient complained of itching, tingli-ng skin of the face, vertigo, nausea, laboredrespiration, spontaneous vision impairment.Objectively: pale face, swollen eyelids andred border, dilated pupils, thready pulse,and rapid labored respiration with crackles.Make the diagnosis:

A. Anaphylactic shockB. SyncopeC. CollapseD. Epileptic attackE. Quincke’s edema

74. A 48-year-old patient came to a denti-st after the maxillectomy on one sideconducted 3 days ago. Remaining teeth arefirm. Treatment plan foresees making anOxman’s denture for the patient. What partof the denture should be produced first?

A. FixatingB. ObturatingC. ResectionD. FormingE. Substituting

75. Parents of an 8-year-old child havemade and appointment with an orthodonti-

st. There are complaints of their child havi-ng traumas of oral mucosa. Objectively:decreased height of the lower face, evertedlower lip, deep labiomental furrow, milkocclusion. The upper incisors fully cover thelower ones; cutting surface of the lower inci-sors make contact with the anterior third ofthe palate. Mesiodistal ratio of the caninesand the first permanent molars is normal.Grouping of the upper and lower front teethis dissimilar. Make the diagnosis accordingto the Kalvelis classification:

A. Deep traumatic overbiteB. Deep incisor overbiteC. Deep neutral occlusionD. Deep prognatic (roof-shaped) occlusionE. -

76. A 46-year-old patient complains ofmastication disorder caused by the lack ofthe 34, 35, and 36 teeth. The antecedentanamnesis is as follows: the teeth wereextracted 3 months ago due to complicationof cariosity. The patient anamnesis: the hi-story of tonsillitis, rheumatoid arthritis andBotkin’s disease. After the appointment wi-th this patient the instruments should besterilized in the following way:

A. Specialized procedureB. Dry-heat sterilizerC. Processing with lysolD. Processing with 0,1% chloramine solutionE. General procedure

77. Preventive examination of a 5-year-oldchild revealed a habit of lower lip biting.What malocclusion may develop if the childkeeps this habit?

A. Anterior biteB. Prognathic biteC. Open biteD. Deep overbiteE. Cross-bite

78. A 7-year-old child has protruding chin,the lower lip overlaps the upper one. Thereare diastema and tremata between the lowerincisors, the lower incisors overlap the upperincisors by 2/3 of the crown height. Fi-rst permanent molars demonstrate Angle’sclass III relation. Sagittal gap is 3 mm. Thecorrect treatment tactics would be to:

A. Use Bruckl’s applianceB. Recommend a complex of myogymnasticexercisesC. Use Angle’s slider applianceD. Use Bynin’s applianceE. Use Schwartz’s appliance

79. A 62-year-old man had been wearinga full removable upper jaw denture. Hecomplains of inability to use this denture

Page 13: Krok 2 STOMATOLOGY€¦ ·  · 2017-10-01Krok 2 Stomatology (англомовний варiант, iноземнi студенти) 2017 рiк 1 1. A 27-year-old man complains of

Krok 2 Stomatology (англомовний варiант, iноземнi студенти) 2017 рiк 11

due to constant burning sensation in thehard palate and dryness of the oral cavi-ty. Objectively: on the hard palate underthe denture base there are an edemaand marked hyperemia. What means ofprevention would be optimal in the givencase?

A. Shielding of denture baseB. Strict adherence to polymerizationprocedureC. Careful selection of impression pasteD. Determination of risk group duringexaminationE. Strict indications for the choice of material

80. A 42-year-old woman has madean appointment with a prosthodonticoffice to make a denture. Objecti-vely: dental formula is as follows:

18 . . . . 13 12 11 21 22 23 . . . . 2848 47 46 45 44 43 42 41 31 32 33 34 35 36 37 .

The patient has deep occlusion; clinicalcrowns are low; equator is not pronounced.The patient suffers from epileptic seizures.What kind of denture should be prescribedfor this patient?

A. Removable partial laminar metal-baseddentureB. Dental bridgeC. Removable partial laminar plastic denturewith retainers (clammers)D. Removable partial laminar denture withsupporting-retaining clasps (clammers)E. Clasp-retained (bugel) removable partialdenture

81. A 27-year-old woman complains of pai-nful and bleeding gums, with the signsaggravating during eating, and indisposition.One week ago she had a case of URTI. Wi-thin the last 5 years she periodicaly presentswith gingival hemorrhages. Objectively: thegums are bright red, markedly swollen; gi-ngival papillae are friable, bleed on the sli-ghtest touch. There is moderate accumulati-on of dental calculus, large amount ofsoft dental deposit. Submaxillary lymphnodes are enlarged, painful on palpation.On X-ray: osteoporosis of alveolar septa isobserved. What is the most likely diagnosis?

A. Exacerbation of chronic catarrhal gingivi-tisB. Exacerbation of initial generalized peri-odontitisC. Hypertrophic gingivitis, edematous formD. Hypertrophic gingivitis, fibrous formE. Acute catarrhal gingivitis

82. A 30-year-old man complains of pain inhis front lower teeth, which he attributesto a trauma to the mental region. Objecti-vely: continuous dentition, orthognathic

occlusion. X-ray shows a median mandi-bular fracture. What dental splint would beoptimal?

A. Flat occlusal splintB. Soldered splint on ringsC. Cap splintD. Weber’s splintE. Plastic mouthguard

83. The department of dentofacial surgeryadmitted a patient who needs repair of apost-traumatic nose wing defect up to 3,0 cmin diameter. The trauma occured six monthsago. What kind of grafting is indicated in thisclinical situation?

A. Grafting with chondrocutaneous flap ofthe auricleB. Grafting with local tissues of nasolabial orcheek regionsC. Grafting with pedicle flap of frontal andbuccal regionsD. Grafting with tubed pedicle flap (Filatov’sflap)E. Free grafting with dermal flap

84. A 25-year-old woman consulted a denti-st about acute pain in her upper jaw on theleft. The pain occurs during eating. Objecti-vely: on the distal approximal surface of the26 tooth there is a cavity filled with lightsoft dentin. Probing causes slight pain alongthe dentin-enamel junction, percussion ispainless. Cold water causes quickly abati-ng pain. What is the most likely diagnosis?

A. Acute median cariesB. Chronic median cariesC. Acute deep cariesD. Chronic fibrous pulpitisE. Chronic deep caries

85. A 27-year-old woman complains ofrecurrent loss of a tooth filling in the lowerright jaw. Objectively: in the 46 tooth onthe masticatory approximal surface there isa defect of hard tooth tissues affecting 1/3of the tooth crown, no tooth discoloration;positive, quickly abating reaction to cold sti-mulus is observed. What denture constructi-on would be optimal in this case?

A. Dental inlayB. Combined crownC. Porcelain-fused-to-metal crownD. Plastic crownE. Partial crown

86. A 10,5-year-old child complains of pai-nful rash on his lips. Objectively: the redborder of the lips is swollen, hyperemic,covered in fissures and numerous scabs ofdried blood. The skin of the upper lip hassmall blisters containing serous substance,which merge with each other in some places.

Page 14: Krok 2 STOMATOLOGY€¦ ·  · 2017-10-01Krok 2 Stomatology (англомовний варiант, iноземнi студенти) 2017 рiк 1 1. A 27-year-old man complains of

Krok 2 Stomatology (англомовний варiант, iноземнi студенти) 2017 рiк 12

Maceration and weeping skin also can beobserved, especially in the corners of themouth. What is the most likely diagnosis?

A. Exematous cheilitisB. Meteorological cheilitisC. Atopic cheilitisD. Actinic cheilitisE. Exfoliative cheilitis

87. A 27-year-old man complains of achi-ng long-lasting pain in the 15 tooth duringeating, especially cold food. Sometimes thepain occurs when the temperature changes.Objectively: on the distal surface of the 15tooth there is a cavity filled with softeneddentin. Probing is painful. Electroexcitabili-ty of the pulp is 35 microamperes. What isthe most likely diagnosis?

A. Chronic fibrous pulpitisB. Acute deep cariesC. Chronic deep cariesD. Hyperemia of the pulpE. Exacerbation of chronic pulpitis

88. A 56-year-old patient suffering fromexacerbation of schizophrenia has beenhospitalised in an oral in-patient departmentwith a diagnosis of the lower jaw displacedfracture in the area of the 34-35 teeth. Whatmethod of treatment should be prescribed?

A. OsteosynthesisB. One arch flat occlusal splintC. Dual splintD. Weber’s dental splintE. Vankevych dental splint

89. During preventive examination a 5-year-old child was found to have insufficientphysiological attrition of tubercles of thedeciduous canines. What treatment tacticsshould a doctor choose?

A. To shave off the retained canine tuberclesB. Medical examination once a month untilthe incisors are replacedC. Medical examination every 6 months untilthe incisors are replacedD. Medical examination every 6 months untilthe canines are replacedE. No medical intervention is necessary

90. A 25-year-old HIV-infected patient cameto a clinic of prosthetic dentistry to havea denture made for him. What aseptic andantiseptic precautions should be taken?

A. According to the schemeB. The impressions should be desinfectedunder a quartz lampC. The patient should be refused appoi-ntmentD. No special precautions are requiredE. The orthopaedist should work in glovesand a mask

91. A 3-year-old child received an injury ofthe upper teeth as a result of a fall. Objecti-vely: crowns of the 51 and 61 teeth areembedded deep into the surrounding tissueswith only their cutting edge visible, the gi-ngival margin is hyperemic, edematic. Whatis the treatment tactics?

A. Tooth extractionB. MonitoringC. RepositionD. Endodontic treatmentE. Anti-inflammatory therapy

92. A 60-year-old patient has been undergoi-ng the procedure of checking the completeremovable dentures construction and fixingteeth on wax bases. The following flaws havebeen detected: fissure between the teeth inthe frontal area and tubercule contact in thelateral area. What mistake had been made?

A. Anterior occlusion was determinedinstead of central oneB. Posterior occlusion was determinedinstead of central oneC. Lateral occlusion was determined insteadof central oneD. Models were plastered in a wrong way inan occluderE. Swabs were crushed, when central occlusi-on was being determined

93. Dental splint is being designed in aprosthodontics clinic for a 39-year-old pati-ent with generalized periodontitis, II degree.Gnathodynamometer is used to measurethe periodontium load resistance. Whatanatomico-functional data are obtained wi-th this method?

A. Masticatory pressureB. Masticatory forceC. Periodontium pliancyD. Masticatory muscles toneE. Masticatory efficiency

94. A 45-year-old man complains of impai-red chewing due to pathologic lower jawmobility. The patient was diagnosed wi-th false joint in the area of absent 33 and34 teeth. On X-ray: defect of the mandi-bular body is 0,8 cm in size. The teethon the fragments on both sides of thedefect are intact. What denture would berecommended in this case?

Page 15: Krok 2 STOMATOLOGY€¦ ·  · 2017-10-01Krok 2 Stomatology (англомовний варiант, iноземнi студенти) 2017 рiк 1 1. A 27-year-old man complains of

Krok 2 Stomatology (англомовний варiант, iноземнi студенти) 2017 рiк 13

A. Oxman’s fixed dental bridgeB. Tigerstedt’s flat occlusal splintC. Weber’s dental splintD. Entin’s stiff head-chin strapE. Tigerstedt’s wire anchor splint

95. A 30-year-old patient complains of painand swelling in the area of the left parotidsalivary gland, which occurred 7 days afterhe had undergone abdominal cavity surgery.Objectively: body temperature equals 39oC,reduced mouth opening; dry mouth; whenthe gland is massaged, there is purulentexudate being secreted from its duct. Thepatient can be diagnosed with the followingdisease:

A. Acute non-epidemic parotitisB. Acute epidemic parotitisC. Phlegmon of submasseteric spaceD. Parenchymatous parotitisE. Phlegmon of parotid-masseteric region

96. A 53-year-old patient complains of anulcer on the lateral surface of the tongue.The ulcer appeared 6 months ago in theresult of a trauma caused by sharp tip of the37 tooth metal crown. A dentist replacedthe crown with the one of better qualityand prescribed keratoplastic drugs. Despitethese measures the ulcer continues to grow.Lately there has been pain during talking,chewing, and swallowing, with occasionalirradiation to the pharynx. Objectively: onthe lateral surface of the tongue there isa painful ulcer with uneven raised densemargins and lumpy floor covered with grayi-sh necrotic coating. What is the most likelydiagnosis?

A. Cancer of the tongue lateral surfaceB. Trophic ulcerC. Traumatic ulcerD. Vincent’s necrotizing ulcerative stomatitisE. Tuberculous ulcer

97. A 15-year-old girl complains oftoothache that persists for a day andincreases on biting. Objectively: in the 36tooth there is a deep carious cavity non-communicating with the dental cavity. Noreaction to the thermal stimuli is observed,probing of the carious cavity floor is painless.Vertical percussion is markedly painful. Gi-ngival mucosa in the area of the 36 tooth isunaltered. X-ray presents with no alterati-ons. Make the diagnosis:

A. Acute serous periodontitisB. Acute suppurative pulpitisC. Acute suppurative periodontitisD. Acute serous pulpitisE. Exacerbation of chronic periodontitis

98. A 35-year-old patient, a veterinarian,came to a dentist with complaints of chi-

lls, fatigue, fever up to 38oC, muscle pain,sensations of dryness, burning, and pain inthe oral cavity, excessive salivation, vesiclesin the interdigital folds, on the lips, oral andnasal mucosa. On examination of the oralcavity there were detected painful bright rederosions with polycyclic contours against thebackground of inflammation. The followingwas observed: scabs on the lips, enlargedtongue, impaired speech, salivation up to 4liters per day. Make the diagnosis:

A. MurrainB. Acute herpetic stomatitisC. ChickenpoxD. MeaslesE. Erythema multiforme

99. During examination of a 5-year-old childthe orthodontist revealed no wear of teeth,no tremata and diastemata, orthogenicocclusion. Which of the following symptomsdetected in the 5-year old child is a sign offuture teeth overcrowding?

A. Absence of tremata and diastemataB. Absence of wear of teethC. Orthogenic occlusionD. Orthognathic biteE. Absence of mesial step in the region ofsecond temporary molars

100. An 8-year-old child has been clini-cally diagnosed with exacerbation of chronicperiodontitis of the 84 tooth. The crown isdecayed by 1/2. What is the optimal tacticsof dental treatment?

A. ExtractionB. Endodontic treatmentC. Endodontic treatment and drug therapyD. Drug therapyE. Opening along the mucogingival fold,drug thrapy

101. A 7-year-old child is diagnosed withchronic granulating periodontitis of the 55tooth. Additionally accompanying diagnosisof rheumatic endocarditis is made. Whattreatment tactics should a dentist choose?

A. Tooth extractionB. Endodontic treatmentC. Endodontic treatment and physicaltherapyD. Case monitoringE. Endodontic treatment and case monitori-ng

102. Parents of an 8-year-old child complainof rashes in the child’s oral cavity. Lately thechild has been inert, refused to eat. On theoral mucosa there are small round erosionswith clear margins. There are vesicles withturbid content on the child’s face and scalp.Make the provisional diagnosis:

Page 16: Krok 2 STOMATOLOGY€¦ ·  · 2017-10-01Krok 2 Stomatology (англомовний варiант, iноземнi студенти) 2017 рiк 1 1. A 27-year-old man complains of

Krok 2 Stomatology (англомовний варiант, iноземнi студенти) 2017 рiк 14

A. ChickenpoxB. MeaslesC. Hypertensive-hydrocephalic syndromeD. Infectious mononucleosisE. Scarlet fever

103. A 45-year-old patient is prescri-bed a dental bridge supported withimplants. In the process of preparati-on to the prosthodontic treatment therewere intraosseous screw two-stage implantsplaced in the area of the 34 and 36 teeth.How long is the period necessary for implantintegration in this case?

A. 3 monthsB. 2 weeksC. 6 monthsD. 10 monthsE. 1 year

104. A 1-month-old child has problems wi-th breastfeeding, insufficiently gains wei-ght. On examination a doctor made thediagnosis of abnormal attachment of thetongue frenulum. What method of surgicaltreatment should be chosen in this case?

A. Transversal dissection of the frenulumB. Lingual frenulectomyC. VestibuloplastyD. Limberg’s relocation of triangular flapsE. Dieffenbach’s frenuloplasty

105. A 5-year-old child complains ofspontaneous pain in an upper jaw tooth onthe right that aggravates at night and duringeating cold food. Objectively: the 65 toothhas a deep cavity communicating with thetooth cavity. Probing is painful, percussion ispainless. Cold water causes long-term pain.What is your provisional diagnosis?

A. Exacerbation of chronic pulpitisB. Acute periodontitisC. Exacerbation of chronic periodontitisD. Acute serous pulpitisE. Acute suppurative pulpitis

106. A 10-year-old girl complains of sensati-ons of dryness and pain in her lips, whi-ch develop in the summer. On examinati-on: the red border has areas of congestivehyperemia and infiltration, scales and scabs,that, when removed, result in hemorrhagi-ng erosions. The skin surrounding lips isunaltered. No rashes are detected on theoral mucosa. What is the most likely di-agnosis?

A. Actinic cheilitis, dry formB. Actinic cheilitis, exudative formC. Atopic cheilitisD. Allergic contact cheilitisE. Meteorological cheilitis

107. A laminar denture for the lower jaw isbeing made for a 54-year-old patient. Baseplate wax is used during the laboratory stagefor wax templates. What group of accessorymaterials does such wax belong to?

A. ModelingB. AbrasiveC. FixingD. ImpressionE. Forming

108. A man complains of short-term painattacks caused by cold stimuli in the tooth onhis upper left jaw, which have been observedfor the last 3 weeks. Objective examinati-on of the 25 revealed a Black’s I class cari-ous cavity located within mantle dentin. Thecavity has narrow opening, its walls and floorare covered with softened dentin. Probingis painful along the dentin-enamel border,percussion is painless, thermometry is pai-nful, the pain quickly abates after removalof a stimulus. Electric pulp test is 6 mi-croamperes. Make the diagnosis:

A. Acute median cariesB. Acute deep cariesC. Chronic median cariesD. Chronic fibrous pulpitisE. Chronic deep caries

109. A 54-year-old man complains of toothhard tissue defects on the upper and lowerjaws. Objectively: in the precervical area ofthe upper and lower premolars within theexternal layer of dentin there are hard ti-ssue defects with surfaces meeting underthe angle. Surface of the defects is smooth,glossy, and dense. What measure should betaken to prevent further progression of themorbid process in the patient?

A. To train him in efficient toothbrushingB. To perform functional examination of thethyroidC. To prescribe calcium preparations intakeD. To limit sour foods in the dietE. To limit sweet foods in the diet

110. A 35-year-old man has been hospitali-zed into a dentofacial unit with complaintsof mobility of the 38, 37, and 36 teeth and afistulous tract in the socket of the extracted35 tooth. The condition has been persisti-ng for 3 months. Insertion of a groovedprobe into the fistulous tract palpatedbared coarse bone fragment that easilymoved under pressure. X-ray of the lowerjaw demonstrates a focus of bone tissuedestruction, with a spot of dense bone ti-ssue 0,5х0,3 cm in size. Make the diagnosis:

Page 17: Krok 2 STOMATOLOGY€¦ ·  · 2017-10-01Krok 2 Stomatology (англомовний варiант, iноземнi студенти) 2017 рiк 1 1. A 27-year-old man complains of

Krok 2 Stomatology (англомовний варiант, iноземнi студенти) 2017 рiк 15

A. Chronic osteomyelitisB. Acute osteomyelitisC. Exacerbation of chronic osteomyelitisD. Chronic periostitisE. Actinomycosis

111. After extreme overexposure to cold a42-year-old patient complains of headachein the left frontal lobe and the left upperjaw. Objectively: the face is symmetrical;left nasal meatus breathing is obstructed,and serous-purulent discharge is beingproduced; palpation of the suborbital areaand further along the mucogingival fold inthe 24 and 25 teeth projection reveals sli-ght pain. Percussion of these teeth is pai-nless. The 24 tooth is filled. The alveolarprocess mucosa has no visible alterations.X-ray imaging shows decreased pneumati-zation of the left maxillary sinus. What is theprovisional diagnosis?

A. Exacerbation of chronic odontogenicmaxillary sinusitisB. Acute periodontitis of the 24C. Exacerbation of chronic periodontitis ofthe 24D. Acute rhinogenous maxillary sinusitisE. Acute albuminous periostitis of the leftmaxilla

112. A 8,5-year-old child is mostly healthy.There is a complaint of pain in the upperleft tooth, due to it having been physicallydamaged 3 hours ago. Objectively: 1/2 of the21 tooth crown is destroyed, the pulp is si-gnificantly exposed, red, sharply painful andbleeding when probed. Percussion of the 21tooth is sharply painful. Choose the optimalmethod of the 21 tooth treatment:

A. Vital amputationB. Vital extirpationC. Devital amputationD. Devital extirpationE. Biological method

113. A 11-year-old child complains of painduring eating food, especially hot, in thelower right lateral tooth. On the masti-catory surface of the 46 tooth there is alarge carious cavity filled with softenedlight-brown dentin. The cavity is located wi-thin parapulpar dentin. In the projection ofmesiobuccal pulp horn the carious cavitycommunicates with the pulp chamber. Deepprobing is painful. Electric pulp test - 60 mi-croamperes. Make the diagnosis:

A. Chronic gangrenous pulpitisB. Chronic hypertrophic pulpitisC. Acute diffuse pulpitisD. Chronic fibrous pulpitisE. Acute focal pulpitis

114. A 28-year-old patient complains of

pain and bleeding of gums in the frontalpart of the upper jaw on the left. Twoyears ago, the 22 tooth was covered witha porcelain-fused-to-metal crown. Objecti-vely: interdental papilla between the 21and 22 tooth is hypertrophied, markedlyhyperemic, overlaps the crown of the 22by 1/3 of its height, bleeds when touched.Periodontal pocket between the 21 and22 tooth is 4 mm deep. Artificial crown islocated on the gingival margin. Radiographyreveals resorption of the interalveolar septabetween the 21 and 22 tooth by 1/3 oftheir height. Specify a priority action in thetreatment of this patient:

A. Removal of the artificial crownB. GingivectomyC. Anti-inflammatory therapyD. Curettage of the periodontal pocketE. Sclerotherapy

115. A 45-year-old patient came to aprosthodontics clinic. During the objectiveexamination the doctor checked the sagittalmovements of the lower jaw. What musclesare responsible for sagittal movements ofthe lower jaw?

A. Lateral pterygoid musclesB. Medial pterygoid musclesC. Mandibulohyoid muscleD. Digastric muscleE. Mentohyoid muscle

116. An 18-year-old patient complains of awhite spot on the vestibular surface of the 21tooth. Objectively: the white spot is locatednear the cutting edge. The spot surface isglossy, its size remains unaltered on drying.Make the diagnosis:

A. Local hypoplasiaB. FluorosisC. Initial cariesD. Enamel necrosisE. Amelogenesis imperfecta

117. A 20-year-old man complains ofspontaneous pain in the 24 tooth, whicharose suddenly and persists for about 15 mi-nutes. Objectively: the distal surface of the24 tooth exhibits a deep carious cavity withoverhanging walls. The cavity is filled withlight softened dentin and communicates wi-th the tooth cavity. The cold stimulus causesacute, slowly abating pain. Percussion causesno pain response. Select the best method oftreatment:

A. Vital extirpationB. Vital amputationC. Biological methodD. Devital amputationE. Devital extirpation

Page 18: Krok 2 STOMATOLOGY€¦ ·  · 2017-10-01Krok 2 Stomatology (англомовний варiант, iноземнi студенти) 2017 рiк 1 1. A 27-year-old man complains of

Krok 2 Stomatology (англомовний варiант, iноземнi студенти) 2017 рiк 16

118. A 25-year-old patient complains of painwhen biting on the 15 tooth. The pain arosetwo days ago, has a constant aching natureand increased significantly over the last day.Objectively: the crown of the 15 tooth is gray,the medial contact surface exhibits a deepcarious cavity communicating with the toothcavity. Percussion causes acute pain, the gi-ngival mucosa in the projection of the 25tooth root apex is hyperemic. The regionallymph node is tender. Radiograph shows anill-defined zone of periapical bone destructi-on. What is the most likely diagnosis?

A. Exacerbation of chronic periodontitisB. Acute serous periodontitisC. Acute suppurative periodontitisD. Chronic granulating periodontitisE. Acute serous periodontitis, intoxicationstage

119. A 27-year-old patient has beenreferred by a prosthodontist for endodontictreatment of the 45 tooth. Objectively: the45 tooth crown is destroyed; the lateralsurface of the tongue and the buccal mucosahave patches of grayish macerated epi-thelium slightly rising above the mucosasurface at the points of direct contact wi-th the 45 tooth. The uvula and palatalbars are stagnant-red in color; hard palatepresents with papulae surrounded with redmargin and covered in grayish epithelium.The submandibular, cervical, supraclavi-cular, and subclavicular lymph nodes areenlarged and painless. What is the provisi-onal diagnosis?

A. Secondary syphilisB. Chronic recurrent aphthous stomatitisC. Lupus erythematosus, patch stageD. Soft leukoplakia (leucoplakia mollis)E. Lichen ruber planus

120. A 43-year-old patient complains ofmobility and significant neck exposure ofthe lower front teeth. Objectively: the gumsin the area of the 44, 43, 42, 41, 31, 32, 33, and34 teeth are pale and cyanotic, non-bleeding.The 42, 41, 31, and 32 teeth exhibit the I-IIgrade mobility. The overcrowding of the 42,41, 31, and 32 teeth is present. The necks ofthe 42, 41, 31, and 32 teeth are exposed by1/2 of the root length, the necks of the 43and 33 teeth are exposed by 1/4. What kindof denture should be applied in this case?

A. Cast removable splintB. Kurlyandsky’s bar splintC. Cap splintD. Partial crownE. Half-ring splint

121. A 35-year-old woman complains oftoothache and thickened body of the mandi-ble. Objectively: the lower left jaw is thi-

ckened, Dupuytren’s symptom is observedon the vestibular surface of the mucogingi-val fold in the area of the 36 and 37 teeth.X-ray of the lower left jaw demonstrates therounded focus of bone tissue destruction wi-th clear margins. The roots of the 36 and 37teeth are resorbed. Puncture yielded brownliquid. What is the most likely diagnosis?

A. Giant cell tumor of the body of mandibleB. Ameloblastoma of the body of mandibleC. Fibrous dysplasia of boneD. Cancer of the body of mandibleE. Sarcoma of the body of mandible

122. A patient complains of the alveolarprocess deformation on the left upper jaw.Objectively: the crown of the 25 tooth isdestroyed with cariosity. X-ray image of theparanasal sinuses shows the left one to haveveil-like shading with clear dome-shapedmargin. X-ray image of the crown of the25 tooth shows absence of the periodontalfissure at the the palatal root apex. What isthe most likely diagnosis?

A. Radicular cyst that invaded in the maxi-llary sinusB. Chronic rhinogenous maxillary sinusitisC. Chronic odontogenic maxillary sinusitisD. Maxillary sinus mucosal cystE. Maxillary cancer

123. A patient with ischemic heart diseasehas developed ventricular fibrillation. Whatis the first-priority therapeutic action?

A. Electric defibrillationB. Lidocaine injectionC. Adrenaline injectionD. Potassium chloride injectionE. Novocaine amide injection

124. A 7-year-old boy came to a dentalsurgeon with complaints of painfulswelling of his right cheek and highbody temperature. Objectively the bodytemperature is 38,2oC, facial asymmetrycaused by the right cheek edema, no skin di-scoloration, skin can be pinched in a fold, nomouth opening reduction. In the oral cavi-ty the mucogingival fold in the area of the84 and 85 teeth is smoothed out, the fold ishyperemic, fluctuation is observed. The 85tooth is destroyed by cariosity, painless onpercussion. Make the diagnosis:

Page 19: Krok 2 STOMATOLOGY€¦ ·  · 2017-10-01Krok 2 Stomatology (англомовний варiант, iноземнi студенти) 2017 рiк 1 1. A 27-year-old man complains of

Krok 2 Stomatology (англомовний варiант, iноземнi студенти) 2017 рiк 17

A. Acute odontogenic suppurative mandi-bular periostitis originating from the 85 toothB. Acute odontogenic albuminous mandi-bular periostitis originating from the 85 toothC. Acute odontogenic mandibularosteomyelitis originating from the 85 toothD. Suppuration of the periapical cyst of the85 toothE. Odontogenic abscess of the buccal areaoriginating from the 85 tooth

125. A 15-year-old adolescent boy complai-ns of pain in the lower right jaw, whichincreases during chewing, and impairedclosure of the teeth. Anamnesis: 2 days agoa trauma was received. Based on objecti-ve examination results and X-ray data thepatient was diagnosed with open fracture ofthe mandible between the 45 and 46 teeth.Choose the method of treatment:

A. Removable bracesB. Temporary immobilizationC. Port’s dental splintD. Rudko’s applianceE. Ligature

126. A woman came to a dentist forconsultation. She is 4 months pregnant.Objectively: marginal gingiva presents withdense gingival papillae, torus-shaped, oval,enlarged up to 1/3 of the tooth crowns. Makethe provisional diagnosis:

A. Hypertrophic gingivitisB. Catarrhal gingivitisC. PeriodontitisD. PeriodontosisE. Ulcerative gingivitis

127. A 53-year-old man complains ofincreased teeth sensitivity to chemical sti-muli. Objectively: the gums are pale pi-nk, roots are bared by 1/3 of their length.Small amount of dental deposit is observed.The 15, 14, and 24 present with cuneiformdefects. Probing of the bared cervices anddefects is painful. What is the most likely di-agnosis?

A. Periodontosis, I degreeB. Catarrhal gingivitisC. Periodontitis, II degreeD. Periodontitis, I degreeE. Ulcerative gingivitis

128. A 24-year-old man complains of pai-nful and bleeding gums. The condition onsetwas 3 days ago after the patient had a caseof acute respiratory disease. Objectivelythe gingival mucosa is swollen, bright-red,bleeds on probing, painful on palpation; ti-ps of the gingival papilla are rounded; softdental deposit is observed. X-ray shows nochanges in the bone. What is the most likelydiagnosis?

A. Catarrhal gingivitisB. Hypertrophic gingivitisC. Atrophic gingivitisD. PeriodontitisE. Ulcerative gingivitis

129. A 22-year-old woman came to a dentistfor preventive examination. During exami-nation of the oral cavity the dentist detecteda defect of hard tooth tissues in the cervi-cal area of the 22 tooth within mantledentin. The dentin is dense and pigmented.No reaction to percussion and probing isobserved. Make the diagnosis:

A. Chronic median cariesB. Cuneiform defectC. Acute deep cariesD. Acute median cariesE. Necrosis of hard tooth tissues

130. Spot X-ray is performed for a 12-year-old child. The X-ray film shows the 35 toothroot to be of nornal length, its walls areparallel to each other and gradually thinout towards the apex. At the entrance theroot canal is slightly smaller in diameterthan near the forming apex. The periodontalfissure is uniformly wide along the wholelength of the formed part of the root. In theapical area of the root it merges with growthzone. What stages of root and periodonti-um development correspond with this X-rayimage?

A. Unformed apexB. Lengthwise growth of a rootC. Open apical foramenD. Unformed periodontiumE. Fully formed root and periodontium

131. A 10-year-old boy complains of painfulsore in the mouth, which has been persi-sting and increasing in size for 1,5 months.Objectively: on the buccal mucosa there isa soft shallow ulcer 2 cm in diameter wi-th uneven undermined edges. The floor ofthe ulcer is tuberous, covered in yellow-gray coating. The ulcer is surrounded withnumerous yellowish tubercles. The regionallymph nodes are elastic, painful, and mattedtogether. These symptoms are characteristicof the following disease:

A. TuberculosisB. Lichen ruber planusC. Necrotizing ulcerative stomatitisD. CancerE. Syphilis

132. A 47-year-old man complains of persi-sting dull pain in his upper right jaw.Anamnesis states that the 13 tooth has beentreated repeatedly to no effect. Objecti-vely: oral mucosa in the area of the 13

Page 20: Krok 2 STOMATOLOGY€¦ ·  · 2017-10-01Krok 2 Stomatology (англомовний варiант, iноземнi студенти) 2017 рiк 1 1. A 27-year-old man complains of

Krok 2 Stomatology (англомовний варiант, iноземнi студенти) 2017 рiк 18

and 14 teeth is hyperemic and painless onpalpation. The crown of the 13 tooth isdestroyed to the gingival margin. Percussi-on is sharply painful. What conductionanaesthesia should be applied to extract the13 tooth?

A. Infraorbital, incisor, palatalB. Infraorbital, incisorC. Infraorbital, palatal, tuberalD. Infraorbital, palatalE. -

133. A 29-year-old man came to anoral surgery department to extract the38 tooth. There are complaints of painand reduced mouth opening. Objectively:body temperature is 38oC, general condi-tion is satisfactory, slight facial asymmetryis observed due to soft tissue swelli-ng under the gonial angle on the left.Inflammatory contracture of the III degreeis observed. In the oral cavity there areedema and hyperemia of mucosa along thepterygomandibular fold on the left. Thesubmandibular lymph nodes are enlargedand painful on palpation. What conductionanaesthesia should be applied prior to theextraction of the 38 tooth?

A. Berchet-Dubov anaesthesiaB. Mandibular, lingualC. Buccal, lingualD. InfiltrationE. Tuberal

134. A 52-year-old man was referred toan in-patient dentofacial department wi-th complaints of pain in the submandi-bular area, aggravating during eating.Anamnesis states frequent exacerbations ofinflammatory processes. Main and additi-onal investigations resulted in provisionaldiagnosis of submandibular sialolithiasis. Aconcrement 1,5 cm in diameter is localizedin the body of the gland. What tactics shoulda dental surgeon choose?

A. Gland extirpationB. Saliva-producing dietC. Physiotherapeutic proceduresD. Puncture biopsyE. Extract the concrement from the glandand place a blind suture

135. A 70-year-old patient complains ofimpaired mastication. Objectively: the 33and 43 teeth on the lower jaw are retai-ned, firm, but have significantly destroyedcrowns. Removable partial denture is requi-red. Choose the optimal method of denturefixation:

A. Extracoronal attachmentsB. Telescopic crownsC. Wire clasps (clammers)D. Cast clasps (clammers)E. Intracoronal attachments

136. A patient diagnosed with rheumatoidarthritis came to a dental surgeon complai-ning of painful mouth opening characterizedby pain on both sides of the jaw. The followi-ng is characteristic of rheumatoid arthritis ofthe temporomandibular joint:

A. Pain is observed on both sides of thetemporomandibular jointB. Pain is observed on one side of thetemporomandibular jointC. Pain can be observed on one or both sidesof the temporomandibular jointD. Clicking is observed in the joint on therightE. Clicking is observed in the joint on the left

137. A 38-year-old man after a domestic acci-dent complains of pain and mobility of hisupper teeth, problems with eating. Objecti-vely: soft tissues edema. The 11 and 21 teethare displaced towards the palate, mobile(II degree), painful on percussion. Mucosasurrounding the affected teeth is hyperemicand swollen. X-ray demonstrates widenedperiodontal fissure of the 11 and 21. Choosethe treatment method:

A. Setting of the teeth and their fixation witha flat occlusal splintB. Extraction of the 11 and 21 teethC. Reimplantation of the 11 and 21 teethD. Immobilization or mouthguardE. Removal of tooth pulp in the 11 and 21teeth

138. Parents complain of painfulness andmobility of the tooth of their 4-year-old chi-ld, which developed after the impact witha wooden object. Objectively: the face isasymmetrical due to swollen tissues of theupper lip. The 51 tooth is intact, with vesti-bular displacement and the II degree mobili-ty, gums around the 51 tooth are hyperemic.What provisional diagnosis can be made?

A. Incomplete dislocation of the 51 toothB. Complete dislocation of the 51 toothC. Contusion of the 51 toothD. Acute albuminous periostitisE. Acute suppurative periostitis

139. A 7-year-old child is in a grave condi-tion, teeth of the upper left jaw are pai-nful, body temperature is 39, 3oC. The chi-ld is pale, adynamic; the face is asymmetri-cal due to infiltration in the upper left jaw.The 64 tooth is filled, painful on percussi-on. The 63 and 65 teeth are intact, painfulon percussion. The I degree of tooth mobili-

Page 21: Krok 2 STOMATOLOGY€¦ ·  · 2017-10-01Krok 2 Stomatology (англомовний варiант, iноземнi студенти) 2017 рiк 1 1. A 27-year-old man complains of

Krok 2 Stomatology (англомовний варiант, iноземнi студенти) 2017 рiк 19

ty is observed; pus is being discharged fromunder the marginal gingiva of the 64 tooth.The alveolar process is deformed at its vesti-bular and palatine surfaces. Make the provi-sional diagnosis:

A. Acute odontogenic osteomyelitisB. Acute albuminous periostitisC. Acute suppurative periostitisD. Ossification periostitisE. Ewing’s sarcoma

140. A 4-year-old child has developedacute spontaneous pain in the tooth on thelower right jaw, which aggravates on biti-ng. Objectively: in the 85 tooth there is adeep carious cavity non-penetrating to thedental cavity. Probing is sharply painful atall points of the cavity floor. Painful reacti-on to cold water stimulus and percussionis observed; mucosa surrounding the 85 ishyperemic. Submandibular lymphadenitis isdetected. Make the provisional diagnosis:

A. Acute pulpitis complicated with peri-odontitisB. Acute albuminous periostitisC. Acute serous periodontitisD. Acute suppurative pulpitisE. Exacerbation of chronic periodontitis

141. An 18-year-old girl came to a dentist tocheck the quality of fissure sealing that hadbeen performed one year ago. Objectively:the sealant is completely retained in the 17,16, 26, 27, 37, and 47. No sealant was detectedin the 36 and 46, in the distal longitudi-nal fissures of these teeth there is softenedenamel. Fedorov-Volodkina Hygiene Indexis 2,5. What tactics regarding the 36 and 46should the dentist choose in this case?

A. Preventive fillingB. Repeated noninvasive sealingC. Invasive sealingD. Applications with fluorine-containing gelE. Electrophoresis of calcium-containingsolution

142. A patient needs the 36 tooth extracted.After administering anaesthesia the doctorstarted applying the elevator. However,immediately after that the patient suddenlypaled, complained of dizziness, ear noise,and blackout and slid down in the chair.What is the most likely diagnosis?

A. UnconsciousnessB. Anaphylactic shockC. CollapseD. ShockE. Hypoglycemic coma

143. A 54-year-old patient complains offrequent crunching sound in the righttemporomandibular joint, which developed

one month ago. In the morning thecrunching is more frequent and decreasestowards the evening. Objectively: the faceis symmetrical, the skin above the joint isunaltered, the mouth opens by 2,9 mm.What is the most likely diagnosis in thiscase?

A. ArthrosisB. Acute arthritisC. Temporomandibular joint dislocationD. Chronic arthritisE. Pain dysfunction syndrome of thetemporomandibular joint

144. A 34-year-old man came to a dentalclinic for extraction of the 26 tooth. Afterapplication of 1,7 ml of Ultracain (Articai-ne) solution for local anaesthesia the pati-ent developed general fatigue and nausea.Objectively: the skin is pale, cold, cyanotic,covered in clammy sweat; BP is 60/40 mmHg. What urgent condition did the patientdevelop?

A. CollapseB. Anaphylactic shockC. Loss of consciousnessD. Bronchial asthmaE. Urticaria

145. Carious cavities of the 11 and 21 teethwere detected during the preventive exami-nation of a 20-year-old patient. What materi-al should be used to fill the detected caviti-es?

A. Microhybrid compositeB. Macrofilled compositeC. AmalgamD. Phosphate cementE. Plastic

146. A 14-year-old boy complains of rapidwearing-off of tooth crowns. Objectively:tooth crowns are worn-off by 1/3. Enameleasily chips off and is pale gray in color.Make the diagnosis:

A. Stainton-Capdepont syndromeB. Dentinogenesis imperfectaC. FluorosisD. Systemic hypoplasiaE. Focal hypoplasia

147. A patient complains of periodical gi-ngival hemorrhages during tooth brushingand increased teeth sensitivity to thermaland chemical stimuli, which persist for thelast 6 years. On examination the gums areswollen and hyperemic. Periodontal pocketsare 5 mm deep with serous purulent content,tooth cervices are bared, I degree toothmobility is observed. On X-ray: irregularresorption of of alveolar septa up to their1/2. What diagnosis corresponds with the gi-

Page 22: Krok 2 STOMATOLOGY€¦ ·  · 2017-10-01Krok 2 Stomatology (англомовний варiант, iноземнi студенти) 2017 рiк 1 1. A 27-year-old man complains of

Krok 2 Stomatology (англомовний варiант, iноземнi студенти) 2017 рiк 20

ven clinical presentation?

A. Exacerbation of generalized periodontitis,II degreeB. Papillon-Lefevre syndromeC. Periodontosis, I degreeD. Exacerbation of generalized periodontitis,I degreeE. Exacerbation of severe catarrhal gingivitis

148. A 63-year-old man complains of painin the area of maxillary mucogingivalfold caused by using a removable lami-nar denture. Objectively: in the area of themucogingival fold there is a trophic ulcer wi-th swollen margins and hemorrhaging floor.Make the diagnosis:

A. Denture-related stomatitisB. Toxic chemical stomatitisC. Toxic infectious stomatitisD. Allergic contact stomatitisE. Greenhouse effect

149. A 64-year-old man complains of painand mobility of his front lower teeth.Objectively: the 43, 42, 41, and 31 teethdemonstrate the III degree mobility. Theseteeth are planned to be extracted andimmediate-insertion denture is to be made.When should this denture be put in place?

A. On the day of the teeth extractionB. In 2-4 days after the teeth extractionC. In 6-8 days after the teeth extractionD. In 3-6 days after the teeth extractionE. In 1-2 days after the teeth extraction

150. During the oral cavity sanation on thevestibular surface of the 21 and 12 teeth inthe cervical area there were detected chalkyspots. Enamel surface is dull and coarse,no reaction to thermal stimuli. Decayed-Missing-Filled Index (dmft/DMFT) is 6,Hygiene Index is 2. Mesial occlusion isobserved. Within the first year of life thepatient had been suffering from frequentcases of URTI and a case of chickenpox. What additional investigation methodswould be useful for the diagnosis-making inthis case?

A. Vital stainingB. Anamnesis dataC. Electric pulp testD. X-rayE. Stomatoscopy

151. A patient complains of pain and sensati-on of heaviness in the left side of his faceand mucous discharge from the nose. Onexamination: left cheek edema, destroyed26 tooth. Tooth percussion is sharply painful.X-ray demonstrates shadowed left maxillarysinus. What disease corresponds with the gi-ven clinical presentation?

A. Acute odontogenic maxillary sinusitisB. Acute rhinogenic maxillary sinusitisC. Chronic odontogenic maxillary sinusitisD. Cyst of the maxillary sinusE. Acute ethmoiditis

152. During application of tuberalanaesthesia the patient developed rapidlyincreasing tissue edema and reduced mouthopening. What resulted in such a condition?

A. Vascular traumaB. Muscle trauma during anaesthesia appli-cationC. Nerve trunk traumaD. Intolerance to the anaestheticE. Anaphylactic shock

153. During application of infraorbitalanaesthesia the patient developed a posti-njection hematoma. What vessel had beendamaged?

A. Infraorbital arteryB. Maxillary arteryC. Pterygoid venous plexusD. Temporal arteryE. Palatine artery

154. A patient came to a dental surgeon withcomplaint of periodical pains in the 22 tooth.On X-ray examination the patient was di-agnosed with granulomatous periodontitisdeveloped due to filling material penetrati-ng the space behind the root apex. Whatfurther treatment tactics should be chosen?

A. Resection of the root apexB. Extraction of the 22 toothC. Prescription of anaestheticsD. Referral to an oncologistE. Recurrent endodontic treatment

155. After the inflammatory process in theparotid area a woman developed frequentpain attacks resembling electric current inher face on the right. The attacks last for 15-20 minutes. The most likely diagnosis is:

A. Trigeminal neuralgiaB. Tympanic plexus neuralgiaC. Trigeminal neuritisD. Exacerbation of chronic maxillary sinusi-tisE. Exacerbation of chronic osteomyelitis

156. A 24-year-old woman came to a denti-st to receive sanation. Objectively on themasticatory surface of the 37 tooth thereis a deep carious cavity connected with thedental cavity. The cavity probing is painless,no reaction to thermal stimuli is observedin the tooth, percussion is painless. EOD is108 microamperes. X-ray shows traces of fi-lling material in the rooth canal of the 37tooth, periodontal fissure is enlarged and

Page 23: Krok 2 STOMATOLOGY€¦ ·  · 2017-10-01Krok 2 Stomatology (англомовний варiант, iноземнi студенти) 2017 рiк 1 1. A 27-year-old man complains of

Krok 2 Stomatology (англомовний варiант, iноземнi студенти) 2017 рiк 21

deformed. Make the diagnosis:

A. Chronic fibrous periodontitis of the 37toothB. Chronic granulating periodontitis of the37 toothC. Chronic granulomatous periodontitis ofthe 37 toothD. Chronic fibrous pulpitis of the 37 toothE. Exacerbation of chronic granulomatousperiodontitis of the 37 tooth

157. A 40-year-old man came to an admi-ssion room with an incised wound of theinfraorbital region received 8 hours ago.On examination the wound underwent pri-mary surgical treatment. In case of an inci-sed wound its edges:

A. Should not be excisedB. Should be closed with secondary suturesC. Should be closed with primary delayedsuturesD. Should be processed with antibioticsolutionE. Should be cleaned and drained

158. An ambulance has delivered an 8-year-old child to an admission room. Anoral surgeon has made the following di-agnosis: odontogenic phlegmon of the ri-ght submandibular area. What surgicalapproach would be advicable for opentreatment of this phlegmon?

A. Dissection in the submandibular area,parallel to the mandibleB. Dissection parallel to the torus mandi-bularisC. Dissection around the mandibular angleD. Dissection along the lower neck foldE. Dissection in the area of pterygomandi-bular fold

159. A 14-year-old child complains ofthrobbing undulating pain in the lowerleft teeth, which aggravates due to hotstimuli. Objectively: on the masticatorysurface of the 36 tooth there is a cari-ous cavity within parapulpar dentin, whi-ch is non-communicating with the dentalcavity. The cavity floor probing is painless,tooth percussion is painful. What treatmentmethod would be optimal in the given case?

A. Vital extirpationB. Devital extirpationC. Devital amputationD. Vital amputationE. Biological method

160. An orthodontist has registered forregular check-ups a 3,5-year-old child,who has a bad habit of finger suckingand presents with infantile swallowing. Onexamination: milk occlusion, direct contact

of incisors. What preventive appliancewould be optimal in this case?

A. Vestibulo-buccal shieldB. Schonherr’s standard vestibular plateC. Frankel’s functional regulatorD. Janssen’s bionatorE. Rudolf’s plate with loops

161. A 45-year-old man complains of painand crepitation in the temporomandibularjoint during the movements of the lowerjaw. Objectively: the face is symmetrical, themouth opens with slight displacement to theleft. Dentition is intact. To clarify the di-agnosis X-ray of the temporomandibular joi-nt was performed. Where should the headsof the mandible be located normally duringmaximum mouth opening?

A. At the top of the articular tubercleB. In the center of the glenoid fossaC. In front of the articular tubercleD. In the center of the articular tubercleE. Closer to the distal part of the glenoidfossa

162. A 45-year-old man complains of painand crepitation in the temporomandibularjoint during the movements of the lowerjaw. Objectively: the face is symmetrical,the mouth opens with slight displacementto the left. Dentition is intact. On occlusi-ography there were detected centric andeccentric supracontacts. What treatmentmethods should be applied in the first place?

A. Selective teeth shavingB. Mouthguard for muscle relaxationC. Appliances that limit mouth openingD. Mouthguards that increase the height ofcentral occlusionE. Lower jaw immobilization

163. A 65-year-old man during thetooth extraction suddenly felt unwell, hedeveloped severe pain irradiating to theleft scapula and numbness of the left hand.Objectively: the patient is pale, beads ofperspiration appeared on his forehead, BPis 170/90 mm Hg, heart rate is 86/min.,rhythmical. The dentist stopped the mani-pulations in the oral cavity. What drugshould be administered in this case?

A. NitroglycerineB. Zelenin dropsC. Valerian tinctureD. ValocordinE. Analgin (Metamizole)

164. A 45-year-old patient after admini-stration of local anaesthesia in preparati-on to oral surgery has suddenly felt unwell,developed increasing edema of laryngealmucosa and respiration disorder. The dentist

Page 24: Krok 2 STOMATOLOGY€¦ ·  · 2017-10-01Krok 2 Stomatology (англомовний варiант, iноземнi студенти) 2017 рiк 1 1. A 27-year-old man complains of

Krok 2 Stomatology (англомовний варiант, iноземнi студенти) 2017 рiк 22

stopped the manipulations in the oral cavi-ty. What type of asphyxia developed in thepatient?

A. StenoticB. DislocationalC. ValvularD. ObturativeE. Aspiration

165. A 19-year-old young man complains ofa fistula on the neck anterior surface, whi-ch periodically reappears at the same place.Objectively: at the neck midline betweenthe hyoid bone and thyroid cartilage thereis a fistula; the skin of the affected area isscarred, drawn-in, and macerated. In thesurrounding tissues a dense band extendingfrom the fistula opening to hyoid bone canbe palpated. A doctor has made a provisi-onal diagnosis of thyroglossal fistula. Specifythe additional method of investigation:

A. Contrast radiographyB. ProbingC. Computer tomographyD. UltrasoundE. -

166. Mother of an 8-month-old girl came toa clinic with complaints of the child’s anxi-ety, fussiness, high fever up to 38, 5oC, si-gns of alimentary canal irritation, vomiti-ng and refusal to eat. On objective exami-nation the child is pale, crying, presents wi-th hyperemia, edema, gingival pain in thefrontal area of the upper jaw, no eruptedteeth can be detected. Make the diagnosis:

A. Hindered tooth eruptionB. Acute herpetic stomatitisC. Food poisoningD. Hematogenous osteomyelitis of the maxi-llaE. Odontogenic osteomyelitis of the maxilla

167. A 45-year-old patient complains ofinability to properly masticate due to theloss of lateral teeth. The 17, 16, 15, 25, 26,27, 37, 36, 35, 44, 45, and 46 teeth are mi-ssing. The retained teeth exhibit the I-IIdegree of mobility. The patient is diagnosedwith generalized periodontitis. Kennedyclass I dentition defects are observed. Whatconstruction would be optimal in the givencase?

A. Clasp-retained (bugel) removable partialdenture with splinting elementsB. Partial laminar dentureC. Elbrecht’s dental splintD. Mamlok’s dental splintE. Cantilever dental bridge

168. Removable complete laminar dentureis being made for a 63-year-old edentulous

man. At the stage of placing the artificialteeth in the dental articulator it is necessaryto determine sagittal articular angle. Thisangle equals:

A. 20-40�B. 5-15�C. 15-20�D. 20-25�E. 40-50�

169. In a new neighbourhood unit of a largecity a dental clinic is being opened. Theclinic will employ 3 prosthodontists. Howmany positions of dental technicians, dentalnurses, and orderlies should be provided?

A. 6 dental technicians, 1 dental nurse, 1orderlyB. 3 dental technicians, 1 dental nurse, 1orderlyC. 3 dental technicians, 1,5 position of adental nurse, 1 orderlyD. 6 dental technicians, 1 dental nurse, 0,5position of an orderlyE. 1,5 position of a dental technician, dentalnurse, and an orderly

170. A 25-year-old man complains of short-term pain in the tooth on the lower ri-ght jaw during eating sweet, hot, and coldfood. Objectively: in the 36 tooth on thedistal surface there is a carious cavitynon-communicating with the dental cavi-ty, dentin is softened. Probing of the cavi-ty floor is painful, percussion is painless.Electric pulp test is 16 microamperes. Makethe final diagnosis:

A. Acute deep cariesB. Acute median cariesC. Hyperemia of the pulpD. Chronic gangrenous pulpitisE. Chronic fibrous periodontitis

171. A 30-year-old woman complains of acarious cavity in the 16 tooth, food retentionin the gap between the 16 and 17 teeth. Duri-ng examination there was detected a cari-ous cavity within mantle dentin with wideopening on the approximal-medial surfaceof the 16 tooth. The cavity floor and wallsare pigmented, dense, painless on probing.Percussion of the 16 tooth is painless. Onthermodiagnostics a short-term reaction canbe observed. Make the diagnosis:

A. Chronic deep cariesB. Chronic fibrous pulpitisC. Chronic median cariesD. Chronic fibrous periodontitisE. Chronic concrementous pulpitis

172. A 40-year-old patient requires surgicalsanation of the oral cavity. Objectively: the36 tooth is completely destroyed. Mouth can

Page 25: Krok 2 STOMATOLOGY€¦ ·  · 2017-10-01Krok 2 Stomatology (англомовний варiант, iноземнi студенти) 2017 рiк 1 1. A 27-year-old man complains of

Krok 2 Stomatology (англомовний варiант, iноземнi студенти) 2017 рiк 23

be fully opened. What anaesthesia would beoptimal for extraction of the 36 tooth?

A. TorusalB. MandibularC. MentalD. InfiltrationE. Berchet-Dubov

173. A 45-year-old woman needs a denture.Objectively: the 17, 16, 15, 14, 12, 25, and26 teeth are missing. Specify the Kennedy’sclass of dentition defects in the given case:

A. II class, 2 subclassB. II class, 4 subclassC. III class, 1 subclassD. III class, 3 subclassE. II class, 3 subclass

174. A 37-year-old patient complains of anaesthetic defect. Objectively: the 13 tooth isdestroyed by 2/3. The tooth is pulpless, theroot canal is filled. How deep should the rootcanal be opened for pivot crown installationin this patient?

A. 2/3 of the root canalB. 1/3 of the root canalC. 3/4 of the root canalD. 1/2 of the root canalE. Full length of the root canal

175. A 10-year-old boy complains of acutepain attacks in the area of his upper leftteeth. The toothache persisted for a night.Objective examination revealed a cariouscavity on the masticatory surface of the 26tooth within parapulpar dentin. Probing issharply painful at all points of the cavityfloor. Markedly positive reaction to coldwater stimulus is observed. Select the mostlikely diagnosis:

A. Acute diffuse pulpitisB. Acute serous periodontitisC. Acute suppurative pulpitisD. Acute suppurative periodontitisE. Acute local pulpitis

176. A 15-year-old patient complains of cari-ous cavity and short-term ”lightning-fast”pain attacks in the 26 tooth. The pain attackscease in 1-2 minutes after eating. Objecti-vely: there is a deep carious cavity filled withsoftened dentin. The cavity floor is painfulon probing. Make the diagnosis:

A. Pulpal hyperemiaB. Acute traumatic pulpitisC. Acute suppurative pulpitisD. Acute local pulpitisE. Acute diffuse pulpitis

177. A 7-year-old girl hit her forehead oneday ago. Several hours after the sustainedtrauma a swelling developed in the left

superciliary area. General condition of thechild is unaffected. Objectively: swelling ofthe forehead tissues spreading towards theleft eyelids; the swelling is soft, fluctuati-on sign is present. Make the preliminary di-agnosis:

A. Hematoma of the left superciliary areaB. Hemangioma of the right superciliary areaC. Fracture of the temporal boneD. Fracture of the frontal boneE. Hematic abscess of the left superciliaryarea

178. Mother of a 10-year-old girl complai-ns of a cosmetic defect of the child’s 22tooth that erupted with damaged enamel.Anamnesis states premature extraction ofthe 62 tooth due to caries complication.There is a white-yellow spot with clearmargins on the vestibular surface of the 22tooth. Enamel retains glossiness, no surfaceroughness can be detected on probing. Makethe diagnosis:

A. Local enamel hypoplasiaB. FluorosisC. Acute superficial cariesD. Chronic superficial cariesE. Systemic enamel hypoplasia

179. A 48-year-old patient has come to ahospital with complaints of defects in theparagingival area and slight sensitivity tothermal stimuli. Objectively: there are hardtissue defects that resemble a wedge withsmooth polished walls on the precervicalvestibular surface of the 23 and 24 teeth.Thermal test is slightly positive. What is themost likely diagnosis?

A. Cuneiform defectB. Enamel necrosisC. Acute deep cariesD. Enamel erosionE. Endemic fluorosis

180. Parents of a 3-year-old child complainthat the child has a neck growth thatdeveloped 3 months after the birth. Objecti-vely: in the upper lateral neck area there is asemicircular neoplasm with limited mobility,soft elastic consistency, no skin alterations,painless on palpation. Puncture yielded pus-like clear yellow substance. Make the provi-sional diagnosis:

A. Branchial cleft cystB. Chronic lymphadenitisC. LymphangiomaD. Specific lymphadenitisE. Hemangioma

181. A 16-year-old adolescent girl complai-ns of pain caused by cold stimuli and foodparticles retained in her upper jaw tooth.

Page 26: Krok 2 STOMATOLOGY€¦ ·  · 2017-10-01Krok 2 Stomatology (англомовний варiант, iноземнi студенти) 2017 рiк 1 1. A 27-year-old man complains of

Krok 2 Stomatology (англомовний варiант, iноземнi студенти) 2017 рiк 24

Objectively: on the contact surface of the24 tooth there is a carious cavity withinparapulpar dentin. The cavity floor and wallsare covered with light softened dentin. Thecarious cavity floor is sensitive to probing,percussion of the 24 is painless. Cold waterstimulus is painful, the pain quickly abatesafter the stimulus is removed. Make the di-agnosis:

A. Acute deep cariesB. Acute median cariesC. Acute diffuse pulpitisD. Chronic fibrous pulpitisE. Chronic deep caries

182. A 15-year-old girl complains of briefpain attacks in her teeth due to chemicalstimuli. Objectively: on the contact surfacesof the 11, 21, and 22 teeth there are enamelareas matt white in color, with lost shine,covered in large amount of dental deposit.Enamel is softened and can be easily chi-pped off with excavator. Probing of lesionsis painless. Percussion is painless. No reacti-on to cold stimuli. Make the diagnosis:

A. Acute superficial cariesB. Acute median cariesC. Acute initial cariesD. Chronic initial cariesE. Chronic superficial caries

183. A 32-year-old patient addressed adentist with complaints of inability to closehis mouth. Objectively the mouth is half-open, the chin is protruding forwards andis displaced to the left. Such conditionoccurred after the mouth was opened wi-de. What is the most likely diagnosis?

A. Anterior right-sided mandibular dislocati-onB. Anterior left-sided mandibular dislocationC. Anterior bilateral mandibular dislocationD. Posterior right-sided mandibular dislocati-onE. Posterior left-sided mandibular dislocation

184. A 37-year-old patient has symmetricalface; the mucosa in the area of the 12 toothroot apex projection is pale pink; palpationis painless; the tooth crown is destroyed by1/3; percussion is painless. X-ray: the rootcanal of the 12 tooth is filled to the apex;granuloma 4 mm in diameter surrounds theroot apex. Choose the method of surgicaltreatment:

A. Granuloma removal with root apexresectionB. Root hemisectionC. Coronary radicular tooth separationD. Root amputationE. Tooth extraction

185. During preventive examination apatient was diagnosed with precanceroushyperkeratosis of the red border of thelower lip. What treatment should be prescri-bed?

A. Surgical removal of the focus withinhealthy tissuesB. Surgical removal of the focus wi-thin healthy tissues + close-focusroentgenotherapyC. No treatment is requiredD. Surgical removal of the focus withinhealthy tissues + chemotherapyE. Palliative treatment

186. A 23-year-old patient is hospitalizedinto a dentofacial unit with provisional di-agnosis of the II degree thermal burns of theright buccal and parotid-masseter region.What scar tissue will develop in this case?

A. Healing without a scarB. Atrophic scarC. Hypertrophic scarD. Hypotrophic scarE. Keloid scar

187. A 56-year-old man complains of painin the gonial angle. Objectively: the 45, 46,34, 35, and 36 teeth are missing; there areslight swelling and reduced mouth openingobserved. X-ray: right-sided fracture of thebody of mandible in the area of the 45 and46 teeth without bone defect. What dentalsplint should be used for treatment?

A. WeberB. LimbergC. TigerstedtD. ZbarzhE. Vankevych

188. A clasp-retained (bugel) removablepartial denture for the lower jaw is to bemade for a 53-year-old patient. Objectively:the 38, 37, 35, 34, 45, 46, and 47 teeth are mi-ssing. The retained teeth are firm, with lowclinical crowns. What fixation method of thedenture would be optimal in this case?

A. Telescopic systemB. Supporting-retaining clasps (clammers)C. Bar systemD. AttachmentsE. Ball joint attachments

189. A 28-year-old woman is diagnosed withchronic generalized periodontitis, II degree.The doctor prescribed her a mouthwash withchlorhexidine gluconate as a part of complextherapy. This drug belongs to the followinggroup of antiseptics:

Page 27: Krok 2 STOMATOLOGY€¦ ·  · 2017-10-01Krok 2 Stomatology (англомовний варiант, iноземнi студенти) 2017 рiк 1 1. A 27-year-old man complains of

Krok 2 Stomatology (англомовний варiант, iноземнi студенти) 2017 рiк 25

A. DetergentsB. DyesC. HalogensD. OxidantsE. Acids and alkalis

190. A 22-year-old patient has suffered uni-lateral linear fracture in the area of thegonial angle. Immobilization was provi-ded with full dental brace with loops andintermaxillary elastic expansion. Recoverywas uncomplicated. The brace should beremoved after:

A. 3 weeksB. 2 weeksC. 1 weekD. 10 daysE. -

191. A 33-year-old man, a metalworker,complains of pain and itching in the gums,gingival hemorrhages intensifying duringtooth brushing. The onset of the diseasewas 1 year ago. Objectively: the gums in thearea of upper and lower frontal teeth arehyperemic, swollen, and cyanotic. There aresignificant mineralized deposits on the teeth;the periodontal sockets are 3 mm deep andproduce small amount of serous discharge.What is the most likely diagnosis?

A. Chronic generalized periodontitis, I classB. Chronic localized periodontitis, I classC. Exacerbation of chronic generalizedperiodontitis, II classD. Acute localized periodontitis, II classE. Generalized periodontosis, I class

192. A 34-year-old man complains of pain inthe area of his right eye, headache, and bodytemperature rise up to 38,6oC. Two days agothe patient developed an infiltration in thelower eyelid of the right eye. Objectivelythe eyelids are markedly swollen, palpebralfissure is closed, conjunctiva is swollen.Exophthalmos is observed. The eyeball isimmobile, vision is impaired. Make the di-agnosis:

A. Orbital phlegmonB. Eyelid phlegmonC. Purulent maxillary sinusitisD. Angular vein trombophlebitisE. Lower eyelid abscess

193. A 35-year-old patient complains ofburns of the face and neck, swelling andburning pain in the affected area. Onexamination: edema of the face and neck,palpebral fissure is narrowed due to swelli-ng, affected skin is hyperemic and coveredwith strained thin-walled blisters filled wi-th clear content. Where blisters are broken,there are pink wounds, sharply painful totouch. Determine the degree of the burns:

A. IIB. IC. III AD. III BE. IV

194. A 19-year-old patient came to adentofacial clinic with complaints of painin the gonial angle on the right, impairedmouth opening and painful chewing. The si-gns had been persisting for 5 days, emergedspontaneously and had been aggravatinggradually. Mandibular contracture is of theIII degree. On examination of the oral cavi-ty: hyperemia, edema of the retromolarspace on the right, hood-shaped mucosafrom under which pus is being dischargedand 2 tooth tubercles can be detected. X-rayshows oblique medial tooth position. Makethe diagnosis:

A. Acute suppurative pericoronitis of the 48toothB. Acute suppurative periostitis from the 48toothC. Chronic local mandibular osteomyelitisD. Fracture of the gonial angleE. Acute submandibular sialadenitis

195. A 45-year-old man came to a dentistwith complaints of a massive and extremelydense (resembling wood) infiltration in theparotid and retromandibular areas, whichpersists for 1,5 months. The patient’s generalcondition remains largely undisturbed, signsof inflammatory process are vague and indi-stinct. Periodically in the infiltration areathe skin assumes cyanotic-purple color, asoft patch appears in the center, where 1-2fistulae develop and discharge pus with whi-te granules. Periodically fistulae close andreopen. Make the diagnosis:

A. Parotid actinomycosisB. Parotid tuberculosisC. Parotid erysipelasD. Chronic parotid sialadenitisE. Adenocarcinoma of the parotid gland

196. Parents of a 6-year-old child complainof their child having a gradually enlargi-ng neoplasm in the left parotid-masticatoryregion. Skin over the tumor is without di-scoloration. The tumor is painless, but whenthe head bends down the tumor increases insize and assumes bluish coloring. What di-sease can be suspected in the child?

A. HemangiomaB. FibromaC. AtheromaD. LymphangiomaE. Cyst of the parotid gland

197. A 40-year-old patient complains of pain

Page 28: Krok 2 STOMATOLOGY€¦ ·  · 2017-10-01Krok 2 Stomatology (англомовний варiант, iноземнi студенти) 2017 рiк 1 1. A 27-year-old man complains of

Krok 2 Stomatology (англомовний варiант, iноземнi студенти) 2017 рiк 26

in the tragus area, clicking sound duringmouth opening, stuffed ears. Objectively:the face is symmetrical, mouth opening pathis straight. Dentition defect can be estimatedas Kennedy I class; the 18, 17, 16, 26, 27,and 28 teeth are missing. In this case theload would be the most traumatizing for thefollowing anatomical structure:

A. Interarticular diskB. Articular capsuleC. Articular headD. Distal slope of the articular tubercleE. Socket floor of the temporal bone

198. A 50-year-old man complains of bareddental cervices on his upper and lowerjaws. Objectively: the teeth and dentitionare intact, clinical crowns are elongated,the teeth have no pathologic mobility, areworn off within the physiological norm.To remove supracontacts it is planned toperform selective teeth shaving. What addi-tional investigation is necessary in the givencase?

A. OcclusiographyB. X-rayC. MasticatiographyD. GnathodynamometryE. Mastication tests

199. A 28-year-old man presents withprofuse caseous coating on the posteriorthird of the back of his tongue, soft palate,

tonsils, and posterior wall of the pharynx.Submandibular, submental and deep cervi-cal lymph nodes have been enlarged for4 months. Two weeks ago the patientdeveloped intermittent fever and generalfatigue. Select the correct sequence of HIVdiagnosing:

A. Enzymoimmunoassay, immunoblotting(Western-Blot)B. CD4 cell count, enzymoimmunoassayC. Complete blood count, viral loadD. Complete blood count, enzymoi-mmunoassayE. Viral cultivation, enzymoimmunoassay

200. A 38-year-old man complains ofsensation of a foreign body on his tongueand development of gag reflex during talki-ng. The signs appeared after the prolongedtaking of antibiotics. Objective examinati-on detected thickened and pigmented fi-liform papillae enlarged to 2-3 cm in si-ze. Histologically papillar hyperplasia andmarked keratinization without alteration ofthe surrounding tissues were detected. Whatis the most likely diagnosis?

A. Black hairy tongueB. Median rhomboid glossitisC. Fissured tongueD. Glossitis areata exfoliativaE. Geographic tongue

Page 29: Krok 2 STOMATOLOGY€¦ ·  · 2017-10-01Krok 2 Stomatology (англомовний варiант, iноземнi студенти) 2017 рiк 1 1. A 27-year-old man complains of

INSTRUCTIONAL BOOK

Ministry of public health of Ukraine (MPH of Ukraine)Department of human recources policy, education and scienceTesting Board

TEST ITEMS FOR LICENSING EXAMINATION: KROK 2. STOMATOLOGY.

Kyiv. Testing Board.(English language).

Approved to print 03.04./№47. Paper size 60х84 1/8Offset paper. Typeface. Times New Roman Cyr. Offset print.Conditional print pages 24. Accounting publishing pages 28.Issue. 297 copies.

Page 30: Krok 2 STOMATOLOGY€¦ ·  · 2017-10-01Krok 2 Stomatology (англомовний варiант, iноземнi студенти) 2017 рiк 1 1. A 27-year-old man complains of

List of abbreviations

A/G Albumin/globulin ratio HR Heart rateA-ANON Alcoholics anonymous IDDM Insulin dependent diabetes mellitusACT Abdominal computed tomography IFA Immunofluorescence assayADP Adenosine diphosphate IHD Ischemic heart diseaseALT Alanin aminotranspherase IU International unitAMP Adenosine monophosphate LDH Lactate dehydrogenaseAP Action potential MSEC Medical and sanitary expert committeeARF Acute renal failure NAD Nicotine amide adenine dinucleotideAST Aspartat aminotranspherase NADPH Nicotine amide adenine dinucleotide

phosphate restored

ATP Adenosine triphosphate NIDDM Non-Insulin dependent diabetesmellitus

BP Blood pressure PAC Polyunsaturated aromaticcarbohydrates

bpm Beats per minute PAS Periodic acid & shiff reactionC.I. Color Index pCO2 CO2 partial pressureCBC Complete blood count pO2 CO2 partial pressureCHF Chronic heart failure pm Per minuteCT Computer tomography Ps Pulse rateDIC Disseminated intravascular coagualtion r RoentgenDCC Doctoral controlling committee RBC Red blood countDM-2 Non-Insulin dependent diabetes mellitus RDHA Reverse direct hemagglutination assayDTP Anti diphtheria-tetanus vaccine Rh RhesusECG Electrocardiogram (R)CFT Reiter's complement fixation testESR Erythrocyte sedimentation rate RIHA Reverse indirect hemagglutination

assayFC Function class RNA Ribonucleic acidFAD Flavin adenine dinucleotide RR Respiratory rateFADH2 Flavin adenine dinucleotide restored S1 Heart sound 1FEGDS Fibro-esphago-gastro-duodenoscopy S2 Heart sound 2FMNH2 Flavin mononucleotide restored TU Tuberculin unitGIT Gastrointestinal tract U UnitGy Gray USI Ultrasound investigationGMP Guanosine monophosphate V/f Vision fieldHb Hemoglobin WBC White blood countHbA1c Glycosylated hemoglobin X-ray RoentgenogramHct HematocritHIV Human immunodeficiency virus